Jump to content

Wikipedia:Reference desk/Science: Difference between revisions

From Wikipedia, the free encyclopedia
Content deleted Content added
Line 579: Line 579:
::Related question: does humanity at present time have the capacity to blow the Moon to smithereens? [[User:Bus stop|Bus stop]] ([[User talk:Bus stop|talk]]) 15:45, 30 November 2010 (UTC)
::Related question: does humanity at present time have the capacity to blow the Moon to smithereens? [[User:Bus stop|Bus stop]] ([[User talk:Bus stop|talk]]) 15:45, 30 November 2010 (UTC)
:::Certainly not. --[[User:TotoBaggins|Sean]] 18:51, 30 November 2010 (UTC)
:::Certainly not. --[[User:TotoBaggins|Sean]] 18:51, 30 November 2010 (UTC)
:All this and yet nobody mentions the late [[Alexander Abian]]. --Anonymous, 23:24 UTC, November 30, 2010.


== Can we blow up comets? ==
== Can we blow up comets? ==

Revision as of 23:24, 30 November 2010

Welcome to the science section
of the Wikipedia reference desk.
Select a section:
Want a faster answer?

Main page: Help searching Wikipedia

   

How can I get my question answered?

  • Select the section of the desk that best fits the general topic of your question (see the navigation column to the right).
  • Post your question to only one section, providing a short header that gives the topic of your question.
  • Type '~~~~' (that is, four tilde characters) at the end – this signs and dates your contribution so we know who wrote what and when.
  • Don't post personal contact information – it will be removed. Any answers will be provided here.
  • Please be as specific as possible, and include all relevant context – the usefulness of answers may depend on the context.
  • Note:
    • We don't answer (and may remove) questions that require medical diagnosis or legal advice.
    • We don't answer requests for opinions, predictions or debate.
    • We don't do your homework for you, though we'll help you past the stuck point.
    • We don't conduct original research or provide a free source of ideas, but we'll help you find information you need.



How do I answer a question?

Main page: Wikipedia:Reference desk/Guidelines

  • The best answers address the question directly, and back up facts with wikilinks and links to sources. Do not edit others' comments and do not give any medical or legal advice.
See also:


November 25

light peak

i have some doubts in light peak topic which and what is the use of the micro controller used in it and how the data conversion takes place (electrical to optical) —Preceding unsigned comment added by 117.254.115.64 (talk) 00:30, 25 November 2010 (UTC)[reply]

Have you read our article on Light Peak and checked the official Intel Research Light Peak Overview website? Nimur (talk) 00:49, 25 November 2010 (UTC)[reply]

The microcontroller probably buffers, handshakes, and maybe adjusts output power. The optical signal comes from infrared lasers. Ginger Conspiracy (talk) 04:12, 27 November 2010 (UTC)[reply]

Does phosphorus (P) really look like that? --Chemicalinterest (talk) 00:58, 25 November 2010 (UTC)[reply]

The two standard allotropes of phosphorus are white phosphorus and red phosphorus; this could be something like "violet phosphorus" or "black phosphorus", both of which are more stable that white or red phosphorus, but require high temperatures and/or pressures to form. It could also be some form of phosphate. Phosphate minerals and Phosphorite are important sources of phosphorus, often for use in fertilizers. --Jayron32 01:07, 25 November 2010 (UTC)[reply]
The only time I recall encountering elemental phosphorus, it looked like white phosphorus. More often, I've encountered calcium phosphate, or various mineral forms like apatite. Nimur (talk) 02:06, 25 November 2010 (UTC)[reply]
My money's on that being a phosphate-containing mineral, rather than pure phosphorous. (Similarly, the samples for 'chromium' and 'magnesium' have the look of metal-containing mineral ores rather than the pure metals themselves — though I'm not qualified to say whether or not they're just very badly oxidized.) TenOfAllTrades(talk) 14:48, 25 November 2010 (UTC)[reply]

Brachistochrone problem

Can anyone explain the Brachistochrone problem in a more simple way than "The curve connecting two points displaced from each other laterally, along with which a body, acted on only upon by gravity, would fall in the shortest time."? Thanks in advance. Toolssmilezdfgsdffgrdsfrtdfh975243 (talk) 01:20, 25 November 2010 (UTC)[reply]

You have two fixed points. How do you connect a wire between them so that a bead moving under gravity gets from the higher bead to the lower bead in the least time? Assume the wire is frictionless. --140.180.14.145 (talk) 01:35, 25 November 2010 (UTC)[reply]
I think the word with is unnecessary, and makes the phrase confusing. 81.131.65.104 (talk) 14:17, 25 November 2010 (UTC)[reply]
I agree. Ginger Conspiracy (talk) 04:07, 27 November 2010 (UTC)[reply]

Corvus

Why were Corvuses so upseting to a ship's balance? Is it because it is a large, heavy mast sticking vertically in the air? If so, couldn't one sail with it lowered? And I can't imagine that it could weigh any more than the ship's mast, or at least not with the leverage the mast would have with such an enormous height. Thanks in advance for the answers. --T H F S W (T · C · E) 02:53, 25 November 2010 (UTC)[reply]

The illustration in the linked article seems to show it being much longer than a mast, and it had to be sturdy enough to support a column of rushing troops, so it would have been both heavy (they gave the estimate of 1 ton) and have had a high center of gravity. Lowering it along the deck of the ship, or better yet, into the hold, would certainly help, but it looks from the diagram like the only way it could be lowered was off the side of the ship, which would pull the ship over to that side (if it wasn't supported by another ship at the far end). StuRat (talk) 03:24, 25 November 2010 (UTC)[reply]
Well, I wouldn't really consider that diagram as an authority, and the beam supporting it is not a mast. And I think corvuses (or whatever the plural for corvus is) went off the front, rather then the side most of the time. And I think a mast would have to be stronger; I don't think a large group of soldiers would have nearly as much weight as a couple tons of sail. Plus, whereas soldiers rushing across are spread out and and are supported at both ends, a mast has to support the hole sail from the top. --T H F S W (T · C · E) 04:29, 25 November 2010 (UTC)[reply]
On a TV show (don't recall which one, but I think it was on the History Channel), they recreated a corvus and found it to be extremely clumsy and dangerous to its deployers. Clarityfiend (talk) 05:02, 25 November 2010 (UTC)[reply]
One other factor to consider is that wood is better under compression (like a vertical mast) than it is under a sheering or bending load (like a deployed corvus). Thus, much more wood is needed to support the 2nd case. Although, while the weight of the sails and the mast itself compresses the mast, the force on the sails, from the wind, is a sheering force, so the mast must be strong enough to withstand that, too (up until the point where they pull in the sails in high winds). StuRat (talk) 17:10, 25 November 2010 (UTC)[reply]
No. The mast is not taking up most of the sheering force of the sail. That is, instead, transferred via the rigging, in particular the stays, shrouds, and sheets. --Stephan Schulz (talk) 00:16, 26 November 2010 (UTC)[reply]
In that case, my point (about the corvus needing to be bulky to support sheering forces not experienced by the mast) is even stronger. StuRat (talk) 04:58, 26 November 2010 (UTC)[reply]
Yes. And the corvus was also intentionally made heavy. It was supposed to smash down any defenders, and to securely fix itself into the enemy ship with a spike. It's not very effective if your armored legionaries run over the boarding bridge while it slips off the other ship...swimming with 40 pounds of bronze armor may be borderline possible for a good swimmer, but it is definitely not a battle-winning strategy. --Stephan Schulz (talk) 10:12, 26 November 2010 (UTC)[reply]
The reason carrying a corvus in rough seas is problematic is that it raises the ship's center of mass. A stable ship will have a low center of mass. The higher a ship's center of mass, the more likely a slight tilt will cause it to capsize. WikiDao(talk) 15:29, 26 November 2010 (UTC)[reply]
I believe everyone here knows that, the real Q is why it raises the center of mass more than the mast does, which we seemed to have answered fairly well. There is also the point about the center of mass being off center, which has also been addressed. StuRat (talk) 17:58, 26 November 2010 (UTC)[reply]
Mast? I had thought the corvus was deployed on oar-ships. Any heavy weight above the water-line, whether in addition to a mast or not, would have made the ship top-heavy, I'm just trying to make that answer clear. WikiDao(talk) 20:00, 26 November 2010 (UTC)[reply]
I thought they had masts for sails and resorted to oars when there wasn't enough wind or it wasn't blowing the right way, but I could be wrong. StuRat (talk) 05:41, 27 November 2010 (UTC)[reply]
At the time in question, warships were primarily oared. They used sails to move around if the wind was favorable, but they would actually take down the masts and sails, and even store them on land if possible, before a battle. One detail of the Battle of Actium was that Mark Anthony's forces, contrary to prevailing practice, took their sails and rigging on board, which decreased maneuverability of the ships and may have been bad for morale. --Stephan Schulz (talk) 17:42, 30 November 2010 (UTC)[reply]

How does radiometric dating work?

I understand the principle behind radiocarbon dating, namely that the fraction of C-14 in the atmosphere is maintained at a constant level by the flux of cosmic rays activating nitrogen. Since organisms exchange carbon with their surroundings, the fraction of C-14 in an organism also remains constant during its lifetime. When the organism dies, C-14 is no longer in steady-state and its fraction begins to decrease, allowing its use for dating.

My question is about other forms of dating, like uranium-lead dating. As far as I understand, uranium concentrations are not being held in steady-state like C-14 is, so all uranium created from the supernova (or equivalent) that formed the solar system should decay at the same pace. If this is so, how can uranium be used to find the age of a rock, and what exactly does this age correspond to? The Wikipedia article on the subject goes in depth on the calculations involved, but not this background. —Preceding unsigned comment added by 68.40.57.1 (talk) 05:48, 25 November 2010 (UTC)[reply]

Quoting from the article you linked to:
Uranium-lead dating is usually performed on the mineral zircon (ZrSiO4), though it can be used on other minerals such as monazite, titanite, and baddeleyite. Zircon incorporates uranium and thorium atoms into its crystalline structure, but strongly rejects lead. Therefore we can assume that the entire lead content of the zircon is radiogenic. Where this is not the case, a correction must be applied. Uranium-lead dating techniques have also been applied to other minerals such as calcite/aragonite and other carbonate minerals. These minerals often produce lower precision ages than igneous and metamorphic minerals traditionally used for age dating, but are more common in the geologic record.
From later parts of the article, it can be more complicated then that as lead can sometimes leach but this seems to be the basics.
Nil Einne (talk) 06:59, 25 November 2010 (UTC)[reply]

Sunrise still becoming later after solstice

I wanted to find when sunrise would be in Cincinnati, so I went to sunrisesunset.com, selected Cincinnati at http://www.sunrisesunset.com/custom_srss_calendar.asp, and got a calendar for December. To my surprise, sunrise is latest at the end of the month: yes, it's only three minutes later than at the solstice, but it's still later. Why would sunrise continue to happen later after the solstice? 66.161.250.230 (talk) 12:29, 25 November 2010 (UTC)[reply]

I think we've discussed this before so you may find something in the archives. In any case [1] [2] should get you started Nil Einne (talk) 14:07, 25 November 2010 (UTC)[reply]
If you look at those data for Cincinnati, you will see that both sunrise and sunset are getting later in late December, and the daylight period is getting longer (as you would expect). The reason is that the moment that the Sun is highest in the sky is not exactly midday, even after accounting for time zones. You can get a better explanation of why in our article on the Equation of Time. Physchim62 (talk) 14:15, 25 November 2010 (UTC)[reply]
That happens because earth's orbit is not a perfect circle. It is slightly elongated. Because of that some days are actually slightly longer or slightly shorter than 24 hours. That causes a slight shift in the sunrise time that compounds the shift in sunrise time due to the change o seasons. The final result is that the latest run rise is slightly shifted from the solistice. 76.123.74.93 (talk) 14:13, 25 November 2010 (UTC)[reply]
People have known about this for a long time, even the ancients who, lacking the distractions of the modern entertainment industry, filled their time with astronomical observations and careful calculations of recurrant celestial events. The Analemma was devised as a means of calculating deviations in the actual day from the mean solar day. --Jayron32 14:58, 25 November 2010 (UTC)[reply]
The earliest sunset is in mid December, the latest sunrise is in early January. http://www.timeanddate.com/worldclock/sunrise.html The shortest day of the year is at the winter solstice. I do hope that the proposed bill coming before parliament in the UK is succesful and we switch to continental-european time so that the evenings are lighter and more enjoyable, particularly in the spring and autumn. 92.28.251.194 (talk) 18:34, 25 November 2010 (UTC)[reply]
If it does, you will find, not immediately but over time, that the nominal times of events start getting later and later, until in the end an equilibrium is reached not much different from if you had done nothing. Ultimately, the times people do things are determined by the Sun. Messing around with the clock, as with daylight saving time, works if you change back and forth every six months, because there isn't time for the adjustment to happen. But it's not going to work to make people get up in the dark on a long-term basis. --Trovatore (talk) 19:32, 27 November 2010 (UTC)[reply]
I infer that you do not work for a living? Because most people's routines are set by their 9 to 5 working hours. 92.15.11.45 (talk) 19:55, 27 November 2010 (UTC)[reply]
Those hours, in the long run, are adjusted to the convenience of the workers (including managers; managers are also workers, and also dislike getting up in the dark). So before too long it'll be 9:30 to 5:30, and then 10 to 6.
This is actually a good example of what I was talking about. When I was a student, doing a summer job for IBM in Tucson, my hours were somewhat flexible, but to match other people I was getting rides with, I went along with the most common schedule, 7 AM to 3:42 PM (the 42-min lunch hour has some historical significance at IBM). It took me a long time to realize why it was so early. Eventually I hit on it: Arizona doesn't observe daylight saving time. So in the summer, people made their own.
That will occur in society at large. --Trovatore (talk) 20:02, 27 November 2010 (UTC)[reply]
I've heard that people start work at an early hour in the US, but in the UK I do not believe that will happen. Being further from the equator, the time of sunrise and sunset is much more variable throughout the year than where you are. In mid-winter on a cloudy day it gets dark at 3.30pm for example, with dawn around 9am. In mid-summer it gets light around 3.30am and dark around 10pm or later. 92.15.14.132 (talk) 20:37, 28 November 2010 (UTC)[reply]
Fine, sure. I didn't mean that particular schedule. I meant that, in the long run, schedules will be determined by solar time, not by nominal time. If you artificially set your clocks an hour later, schedules will eventually be an hour later by nominal time. Not necessarily exactly — different organizations will respond differently — but that's the best first-order estimate. --Trovatore (talk) 01:14, 29 November 2010 (UTC)[reply]
I used to do office work on flexitime: some people arrived and left early, but that was due to wanting to avoid the rush hour crowds; others used to start and leave late. The statistics show that most people do fixed hours. The point of switching to European continental time is the longer-lit evenings allowing more and pleasanter outdoor recreation in the evenings, plus the syncronisation of business hours. 92.24.176.72 (talk) 13:45, 29 November 2010 (UTC)[reply]
But even "fixed" hours will change (possibly, to different fixed hours). You can't change people's preferences for when they get up, work, sleep, etc, just by putting a different number to it. The current equilibrium, ultimately, is driven by the hours of sunlight. Put a different number to the same hours, relative to sunrise and sunset, and the hours that people do those things will likewise change. --Trovatore (talk) 20:50, 29 November 2010 (UTC)[reply]
Are you not forgetting the effect of electric light on society? The proposed change is an optimisation of that. You should try getting a job in the UK - then you will rapidly drop these fantasy pastoral ideas. If you are an employee and not on Flexitime then you cannot choose. People in the UK are accustomed to getting up or starting work before dawn in winter. Perhaps in the backwoods of the US, where there is little contact with the rest of society, your ideas are true, but not in the typical urban culture. If what you are suggesting is true, then business hours would change with the seasons (particularly in the north where the UK is) but they do not. 92.29.120.7 (talk) 11:09, 30 November 2010 (UTC)[reply]
I explained why business hours don't change with the seasons. It's because seasons are too short. Let a season go on long enough, and they would indeed change.
The hour on a clock is just a number. In the long run, the equilibrium is not determined by that, but by sunlight. And no, I'm not forgetting about electric lights. --Trovatore (talk) 19:07, 30 November 2010 (UTC)[reply]
Lots of places I have worked in the UK have offered me flexibility of about half an hour, so I could work 8:30-4:30, or 9-5, or maybe even 9:30-5:30. Loads of desk jobs work like this, and times shift depending on many factors. Lots of offices use hours slightly offset from 9-5. If people wanted to start and finish such jobs earlier, they could. People are just as likely to start them later as they adjust. But these are desk jobs. Many, many people in the UK work shift jobs that do not run 9-5: they do not currently experience the light mornings/dark evenings that you and journalists complain of, nor will they win lighter evenings as a result of a change to continental time. Although we could then declare 1am (which would really be midnight) to be the beginning of a new day, solving the illogicallity another user was complaining about :) 86.161.109.130 (talk) 01:19, 29 November 2010 (UTC)[reply]
I raised this same question earlier this year and received some excellent answers. See HERE. Dolphin (t) 01:15, 26 November 2010 (UTC)[reply]

Special relativity

It's been said that special relativity raises the status of measurement - that the value of a quantity is intimately tied to how it can be measured. But where in special relativity does that become important. I learned SR by first having the Lorentz transformations derived, and (almost) all the results followed from these equations. So where does the elevation of measurement come into effect? 70.52.44.192 (talk) 13:02, 25 November 2010 (UTC)[reply]

Are you sure that the statement refers to special relativity, not quantum mechanics? --Wrongfilter (talk) 14:34, 25 November 2010 (UTC)[reply]
How did you derive the Lorentz equations? I've certainly seen them derived starting with a discussion about how time and space intervals are measured e.g assuming distances are measured by sending and receiving light signals. As I recall the term "operational definition of measurement" was thrown about. 129.234.53.175 (talk) 18:19, 25 November 2010 (UTC)[reply]
The Lorentz equations are so basic that the way you derive them depends a lot on which assumptions you want to use. It's kind of like proving 2+2=4.
I don't think I agree that special relativity changes the nature of measurement, versus Newtonian physics. What I do think, though, is that physicists and mathematicians overlooked special relativity for decades because they weren't thinking carefully about measurement. That was the only real contribution of Einstein's 1905 paper. the Lorentz transformations had been derived already (by Lorentz) but people were still stuck to Newton's idea of absolute mathematical time, which they believed was the thing being measured even if physical clocks were affected in such a way that they measured it wrong. Einstein finally threw that away, and people who read his paper finally realized why the Lorentz transformations made sense. It's interesting that the key to understanding quantum measurement turned out to be the same as the key to understanding special relativity: treating the measurement apparatus as a physical system that follows the same rules as the thing being measured. I have the fond hope that whatever is preventing us from understanding quantum gravity will turn out to be equally fundamental... -- BenRG (talk) 21:35, 25 November 2010 (UTC)[reply]

Relative velocity

I think it is more practical saying Lorentz assumed the Lorentz Transformation (LT) and Einstein is the first person trying to derive it. If you look at the time equation of LT, you will wonder how did Lorentz get it? But after you change the spatial equation to x=(x'/γ)+vt, and change the time equation to t=(t'/γ)+(vx/c^2) then put the right part of the equation into t of the previous equation, you will find out that the result is x = γ(x'+vt'). Yes, after you combined equations in LT you get the spatial equation of the inverse LT. Do you know how Lorentz get the time equation? It is so simple that Lorentz just assumed the "hypothesis of ruler contraction" to get x'= γ(x-vt), and the x= γ(x'+vt') for inverse LT, then replaced the x' in the later equation by γ(x-vt); and ha, there is the time equation in LT. Any two of the four equations in LT and inverse LT can derive the other two of them. That means, no matter how people derive LT, the equations of LT and inverse LT will always coexist. Logically speaking, we may assume S' is moving at velovity v and S is rest or we may assume S' is rest and S is moving at velocity -v but to assume both conditions to coexist we should be able to find something very bad in the LT. It is bad, in LT, the v is always zero.Jh17710 (talk) 04:29, 26 November 2010 (UTC)[reply]

The phrases "is at rest" and "is moving at velocity v" are meaningless, unless it's clear what movement is being measured relative to. If the x and x' axes point in the same direction, then the statements "S' is moving in the x direction relative to S at speed v" and "S is moving in the x' direction relative to S' at speed –v" mean the same thing. There is no inconsistency there, and no problem with the LT and the inverse LT coexisting in the same problem. Red Act (talk) 19:40, 26 November 2010 (UTC)[reply]
It is not easy to see the problem by looking at the four equations only. However, if you add two equations t'=t/γ and t=t'/γ that we could derive within LT and inverse LT, then you will see the problem very clearly. We will have γ=1 and so that v=0. How do we derive t'=t/γ and t=t'/γ within LT and inverse LT? Please refer to http://en.wikipedia.org/wiki/Wikipedia:Reference_desk/Archives/Science/2010_November_19 .Jh17710 (talk) 05:28, 27 November 2010 (UTC)[reply]
I've updated the page you linked to with explanations of your latest errors. Red Act (talk) 08:09, 27 November 2010 (UTC)[reply]

Platinum arsenide

Does anyone here know how to extract platinum metal from platinum arsenide (in my case it is sperrylite) and disposing the arsenic safely? I have search the internet and found no information on this.

Another thing I want to know that when we burn platinum arsenide,does it decompose? I gues it is because platinum is noble metal,and I have done and experiment,when I burn it,it turn to black and somewhat seem shrinked. If it does,it will decompose to platinum metal rather than its oxides?

I heard that platinum arsenide is attacked with oxidizing acid like nitric acid, when it attacked,does it form platinum nitrate and arsenic nitrate or just eat the arsenic away leaving the platinum metal?

Thank you very much! —Preceding unsigned comment added by 124.82.11.255 (talk) 14:08, 25 November 2010 (UTC)[reply]

Chemically extracting metals from their ores is actually usually a very dangerous process. Platinum extraction, as you note, produces nasty arsenic compounds. Gold extraction often involves the use of cyanides, see Gold cyanidation. Metal extraction is almost universally poisonous, environmentally destructive, and/or energetically expensive. See tailings and slag for examples of toxic wastes from various stages of metal extraction processes. --Jayron32 15:02, 25 November 2010 (UTC)[reply]
aqua regia will convert this to chloroplatinic acid H2PtCl6, and nitric acid to Pt(NO3)4. Burning the arsenide should remove the arsenic as an oxide vapour, but this could be polluting. Graeme Bartlett (talk) 21:17, 25 November 2010 (UTC)[reply]

Will the reaction with nitric acid produce arsenic nitrate?

ECG

Performing ECG on a patient having a metalic pin in his femurKhuloodm (talk) 16:12, 25 November 2010 (UTC)[reply]

Dose a metalic pin in a patient's bone affect ECG readingsKhuloodm (talk) 16:22, 25 November 2010 (UTC)[reply]

I have no idea, having a look at the Electrocardiography article, I can't deduce any plausible reason why a metal pin should affect ECG readings, but if that's a homework question, maybe it's one of those trick questions that only ECG technicians know the answer to. Vespine (talk) 21:44, 25 November 2010 (UTC)[reply]

No, a conductive metal pin would not influence a ECG any more than normal conductive saline fluids. MRI is a different story, but femoral traction pins should be safe at 1.5 Teslas: PMID 19897988; unlike other kinds of implants at higher magnetic strength: PMID 16835741. If this is an for an actual patient, please don't ask the reference desk. If it's for homework, you shouldn't really be asking it here either. Ginger Conspiracy (talk) 02:25, 27 November 2010 (UTC)[reply]

Human urine

What does human urine taste like ? Obviously I have no interest in trying it out and I'm not asking you to go do some original research I'm just morbidly curious. I'm more looking for scientific answers in the form: human urine contains these compounds, which are also found in these more commonly ingested substances, so it might taste like this. Thanks. 24.92.78.167 (talk) 17:29, 25 November 2010 (UTC)[reply]

Our article on urophagia discusses this a bit - it seems to vary quite a bit based on what the excreter has recently eaten and drunk. Because salts are concentrated in it (per our article), I assume it is usually salty unless something odd is affecting the flavour/odour. Matt Deres (talk) 18:33, 25 November 2010 (UTC)[reply]
It's definitely salty. It tastes pretty much how it smells: like urine. Urine is generally sterile, and safe enough to drink in small quantities; there's no reason not to try some (other than the grossness factor, which isn't insignificant). Buddy431 (talk) 00:03, 26 November 2010 (UTC)[reply]
Its salty (due to the salts) and has a bitter / ammonia-like taste due to the urea. Beyond that, it very much depends on what else the person has been eating drinking. Rockpocket 00:13, 26 November 2010 (UTC)[reply]
In the early days of medicine, physicians would taste the urine of any patient suspected of suffering diabetes. If the urine was slightly sweet due to the presence of sugar this confirmed the diabetes. See History. Dolphin (t) 01:05, 26 November 2010 (UTC)[reply]
You're clearly referring to diabetes mellitus, as distinguished from diabetes insipidus (the latter associated with dilute, not sweet, urine). Note that urine contains no sugar until the diabetes mellitus is sufficiently advanced for blood glucose to exceed the renal threshold (about 180 mg/dL or 10 mM), so the "taste test" would not rule out early diabetes mellitus (of course, diabetes mellitus generally isn't symptomatic until the renal threshold of glucose is exceeded, so it was a useful test). -- Scray (talk) 01:44, 26 November 2010 (UTC)[reply]
Mellitus means sweet in Greek... that's an intentional correlation. Apparently a number of cultures either tasted the urine themselves or used animals (ants or bees) to determine whether or not a particular individual's urine contained sugar, making it sweet. Shadowjams (talk) 12:25, 26 November 2010 (UTC)[reply]
Much of taste comes from smell. 66.108.223.179 (talk) 23:06, 27 November 2010 (UTC)[reply]
Actually, much of flavor comes from smell, taste is pretty much specific to the taste buds. Rockpocket 17:34, 28 November 2010 (UTC)[reply]

cloning plants

Were scientist cloning plants before Dolly the Sheep? If so, for how long? —Preceding unsigned comment added by 69.247.48.131 (talk) 20:47, 25 November 2010 (UTC)[reply]

You don't need to be a scientist to clone plants. Everytime you take a cutting of a house plant, you are cloning it! 86.162.106.18 (talk) 21:07, 25 November 2010 (UTC)[reply]
King's Holly Mac Davis (talk) 23:16, 25 November 2010 (UTC)[reply]
Banana's are famously propagated by cloning. Bananas have probably been cultivated for the past 7000 years in some areas [3], but it's unclear how long they have been cloned, rather than grown from seed. In any case, grafting has been practiced in the far east for the past 4000 years or so. Buddy431 (talk) 00:12, 26 November 2010 (UTC)[reply]
The Navel Orange is my favorite example, since all of the millions and millions consumed over the last 180+ years have been, essentially, the same orange! The Masked Booby (talk) 03:30, 26 November 2010 (UTC)[reply]
Along these lines, the same can be said for every commercial apple variety. There's only one granny smith, only one red delicious, etc. The point is that if a certain apple tree makes tasty apples, there is no guarantee that the offspring grown from seed (indicating sexual reproduction) will taste the same. The only way to ensure this is by grafting (cloning) one tree over and over. SemanticMantis (talk) 15:13, 26 November 2010 (UTC)[reply]
That's not strictly true: as the Red Delicious article points out, there have been a number of mutations that have occurred over the last hundred years, leading to multiple strands (genetically similar, but not identical), that may truthfully be called Red Delicious apples. In general, though, you are quite correct. Buddy431 (talk) 23:32, 26 November 2010 (UTC)[reply]
Thanks for the clarification Buddy431. Wouldn't we expect the same state of affairs for navel oranges and bananas then? Or do you think that the other fruits have not (for some reason) mutated over myriad cycles of cloning? SemanticMantis (talk) 23:13, 29 November 2010 (UTC)[reply]
For naval oranges, definitely. See this list of orange types. I count many dozens of varieties. Note that in most cases, the first tree where the mutation occured is identified, from which all trees of that variety were cloned. I'm not sure about the Cavendish banana; it's only been cultivated heavily for the last 60 years or so, leaving less time for mutations than red delicious apples or navel oranges. Still, we'd expect some mutations, but I can't find any list of varieties like you can for red delicious apples or navel oranges. Buddy431 (talk) 03:21, 30 November 2010 (UTC)[reply]

Research subject

Is there a way I could get paid to become a psychological/cognitive research subject? I have a lot of strange abilities. I lucid dream every night, I can depersonalize, derealize, and überrealize very easily and at will. Psuedohallucinations are common, and so are false memories, illusions of precognition, and many other little oddities. None of these things have ever bothered me in life. I enjoy them and learning how to manipulate them. I would like to learn to do more of these things, and help humanity's understanding of these mental effects. I am also very interested in being injected with psychoactive drugs during the dream state to see how sensations and perceptions are altered. My dream recall can go very deep, and is easily trained and untrained. I would really like to find a situation where I can spend some time delving into my own mind in waking and dream states while free from the responsibilities of civilization. Any ideas? -- Mac Davis (talk) 23:33, 25 November 2010 (UTC)[reply]

Its unlikely anyone is going to inject you with psychoactive drugs, but this site should give you some pointers of how to get involved in dream research. Rockpocket 00:18, 26 November 2010 (UTC)[reply]
Hey! That's a really good site. I read a lot of it. I also found a presentation from DEF CON with some scripts and schematics. I also already have somebody reliable to work with. —Preceding unsigned comment added by Mac Davis (talkcontribs) 04:12, 26 November 2010

You can look on the bulletin boards around your local college or university psychology department to see who needs research subjects; they often pay nominal fees for your time. But please don't specifically seek out experiments involving areas in which you think you may be a statistical outlier. To do so will skew the experiments' results. Ginger Conspiracy (talk) 01:37, 27 November 2010 (UTC)[reply]

Car turbochargers and torque

What causes turbos to produce so much torque (especially smaller twin turbos)? They smaller twin turbos are always capable of producing so much more torque than horsepower. Why is that? —Preceding unsigned comment added by 76.169.33.234 (talk) 23:38, 25 November 2010 (UTC)[reply]

Your statement that some turbos are capable of producing more torque than horsepower is meaningless. Torque and power are two different quantities. Torque is typically measured in Newton.metres and power is measured in Newton.metres.second-1, Watts, kilowatts or horsepower.
Consider an engine producing a torque of 1000 N.m. If the speed of this engine is 1000 RPM it is producing power of 104.7 kW, but if the speed is 10,000 RPM it is producing 1047 kW.Dolphin (t) 04:58, 26 November 2010 (UTC)[reply]

Let me use a specific example to rephrase the question in a way that the OP actually means and would be helpful to follow-up RefDeskers: Why do turbo-charged engines such as that in the SEAT Leon Cupra (2L Turbo, 177kW @ 5,700-6,300rpm, 300Nm @ 2,200-5,500rpm) produce so much more torque so much lower down in the rev range than naturally aspirated engines of the same power e.g. the Honda S2000 (2L NA, 177kW @ 8,300rpm, 208Nm @ 7,500rpm)? The torque curves of turbo charged engines are disproportionately swelled at lower RPMs as compared to NA engines. What makes turbochargers so naturally effective at lower RPMs? Zunaid 10:05, 26 November 2010 (UTC)[reply]

Thanks for that clarification. It was perfectly clear to me what was being asked; I don't know if Dolphin really didn't understand or was just being difficult. StuRat (talk) 17:51, 26 November 2010 (UTC)[reply]
See:Turbocharger--Aspro (talk) 17:16, 26 November 2010 (UTC)[reply]
Does that article actually answer the question ? If so, I must have missed it. StuRat (talk) 17:55, 26 November 2010 (UTC)[reply]
Yes. More air drawn into the cylinder on the intake stroke.--Aspro (talk) 18:01, 26 November 2010 (UTC)[reply]
And how does that alter the HP to torque ratio ? StuRat (talk) 23:01, 26 November 2010 (UTC)[reply]
More oxygen availability causes a faster burn with the same amount of fuel, and possibly more importantly for the low revs case, allows a complete burn with more fuel. Ginger Conspiracy (talk) 02:36, 27 November 2010 (UTC)[reply]
Ginger Conspiracy has written More oxygen ... with the same amount of fuel ... If so, that would be a higher air-fuel ratio and I would disagree with that. Use of turbo-charging does not require, or warrant, a change in air-fuel ratio. The primary effect of supercharging and turbo-charging is to significantly increase the volumetric efficiency - more air and fuel per intake stroke, and therefore more heat released during each power stroke; so greater torque and greater power. An increase in volumetric efficiency can cause a 1 litre engine to have the output of a 2 litre engine - greater torque, greater power and greater fuel consumption at the same engine speed. Dolphin (t) 05:26, 27 November 2010 (UTC)[reply]
I don't think the above answers explain why one gets more torque at lower RPM than an equivalently powered NA engine. A turbocharged 1L producing the power of a 2L (say 110kW) will more than likely produce the torque of a 2.4L (say 240Nm). The torque boost is always disproportionately greater than the power boost when compared to equivalent NA engines, and always at lower RPM. It seems that turbos boost more effectively at lower RPM. Why? Zunaid 16:22, 27 November 2010 (UTC)[reply]
What makes turbochargers so naturally effective at lower RPMs?
That been answered. If you were to ask why turbo's don't show the same proportion of torque at the upper end as NA . The answer would be that if they were allowed to, the stresses imposed upon the components would shorten engine life. --Aspro (talk) 20:40, 27 November 2010 (UTC)[reply]
Turbocharged airplane engines, and possibly turbocharged car engines, are equipped with wastegates solely for the purpose of regulating the boost of inlet manifold pressure. The higher the engine speed the more exhaust gas is diverted away from the turbocharger to reduce the effectiveness of the turbocharging and limit the speed of the turbine. If the inlet pressure was not limited by the wastegate the turbine could overspeed itself to destruction, and there would be increasing potential for detonation at higher engine speeds. If turbocharged engines in cars also have wastegates then that explains why the effectiveness of the turbocharging is so significant at low engine speed, but less so at higher engine speeds. Also see Turbocharger#Wastegate. Dolphin (t) 00:31, 28 November 2010 (UTC)[reply]


November 26

Swallowing one's tongue

I read the following sentences on the Wikipedia: "Teammates Ivica Dragutinović and Andrés Palop immediately ran to his side as he lost consciousness. Moments later, club medical staff and other players followed suit, as Dragutinović stopped Puerta from swallowing his tongue." What is "swallowing ones tongue"? Can you give some explanations? Thank you! —Preceding unsigned comment added by 72.198.195.96 (talk) 05:43, 26 November 2010 (UTC)[reply]

When you lose consciousness your tongue can lose muscle tone and (especially if you're flat on your back) can fall back in your mouth occluding the airways. It's not that you actually 'swallow' it in the way that you swallow food, but merely that if can block the airway. That's why one of first steps in first aid with an unconscious patient is to turn them on their side (along with preventing choking on things like spontaneous regurgitation). --jjron (talk) 08:24, 26 November 2010 (UTC)[reply]

RFID

   Can RFID tagging (i.e. of products exposed for sale in a retail store) be used to defeat aluminium-lined bags used by shoplifters? In addition, what do RFID scanners which are to be placed at a store's entrance look like; are they conspicuous enough to deter would-be shoplifters from even trying? Alternatively, can RFID scanners placed at store entrances be made inconspicuous enough to aid in apprehending shoplifters? Rocketshiporion 05:50, 26 November 2010 (UTC)[reply]

Aluminum foil will attenuate RFID signals in the same way it attenuates most security tags. The amount of foil required to prevent a detection will depend on the details of the system, but for many RFID uses the signal may be undetectable with only 1 to a few layers of foil. Dragons flight (talk) 07:30, 26 November 2010 (UTC)[reply]
Note that current security measures try to detect an object in the vicinity of the door, but a better approach is to continuously monitor the position of every item in the store, and alarm when any of them disappear, noting the site of the disappearance. This requires greater range, bandwidth, and computing power to track all those objects, but it is possible right now. However, the cost of the system and size of the tags makes it not yet practical for most items. Perhaps very expensive items, like jewelry, might be the first to get this treatment. StuRat (talk) 17:46, 26 November 2010 (UTC)[reply]

mining asteroids

Hi, I came across this page where they say an asteroid (3554 Amun) that is a mile wide contains 30 times as much metal as Humans have mined throughout history (ever). Surely this is wrong, or am I understanding it incorrectly? Sandman30s (talk) 06:45, 26 November 2010 (UTC)[reply]

I agree it is wrong, though perhaps not as wrong as one might guess. According to list of countries by iron production, global iron mining was 2.3 billion tons / year in 2009. As raw metal, that would have volume of 0.3 km3. 3554 Amun is only about 7 times that annual volume. So definitely not 30 times all metal ever, but still a large amount on the scale of iron mining. Also, iron stands out for its very large production volumes. Most other metals we mine are in much smaller quantities (e.g. copper and aluminum are only a few percent of the iron values), so a concentration of those metals would be comparatively more significant if one existed. Dragons flight (talk) 07:50, 26 November 2010 (UTC)[reply]
Thanks, I would never have imagined those figures... Sandman30s (talk) 09:48, 26 November 2010 (UTC)[reply]
Hang on a moment. I could not find a web page giving the volume of Amun (I did find multiple sources giving its "diameter", but since small asteroids are not spherical, this gives little idea of its volume). But Wikipedia's page shows its mass as 1.6e13 kg, which is 16 billion metric tons. This accords with Dragon's figure of 7 times the 2.3 billion tons of iron mined in 2009, if those are metric tons; if they're short tons, as one might expect from a US source, it would be nearer 8 times. But several other Internet sources give Amun's mass as 30 billion metric tons, which (if iron) would be equivalent to 13 or 14 years' terrestrial production rather than 7 or 8. Still nowhere near 30 times the world's all-time production; perhaps someone slipped a factor of 1,000 in their original calculation. --Anonymous, 02:02 UTC, November 26, 2010.
Hangh on another monent, list of countries by iron production actually gives iron ore mining statistics, not refined iron production. Thar page needs to be fixed. 75.41.110.200 (talk) 05:01, 27 November 2010 (UTC)[reply]
Good catch! I've put a "needs expert attention" flag on it. Still, judging by the some of the numbers mentioned in one of the linked references, the proportion of iron in iron ore is often pretty large, between say 25% and 40%, so this only introduces an error of a factor of 3 or so. Maybe something like 40 years' production, then. --Anonymous, 05:50 UTC, November 27, 2010.
Hmmm, that's 40 years CURRENT production. If production grows at 7% a year which is not unreasonable to assume, this equals to a doubling of production every decade. I don't know specifically about iron, but this is what has been happening to Oil for over a century. If production doubles every decade, it means that in the last ten years, you have actually produced more resource then ALL THE PRECEDING production combined. Think about it: If you mined one "unit" of iron in the 1st decade, and you mined 2 units in the next decade, in the decade after that you'll mine 4 units, this is more then 1 and 2 combined. The next decade you will mine 8 units, this is more then 1, 2 and 4 combined. And so on. So saying 40 years of current production could very easily be orders of magnitude more then what has ever been produced, if the rate of growth is just 7% annually. Scary but true. Hubbert curve, Hubbert peak theory, Peak oil. Vespine (talk) 23:30, 28 November 2010 (UTC)[reply]
Perfectly true, but we don't hear people talking about "peak iron", so I doubt there has been such a rate of increase. Data? I couldn't readily find anything by googling. --Anonymous, 03:17 UTC, November 29, 2010.
I wasn't trying to say anything about peak iron specifically. My main point is that even with growth rates of 7% a year which is not considered "aggressive" in our consumerist culture, 40 years worth of current production can easily be far more then has ever been produced in the past, regardless of how much time is "in the past" or the resource. Vespine (talk)
Right sorry, i think i misunderstood your question a little, this article has a LOT of data which I find a bit confusing because it talks about "exports" and all sorts of other "percentages" which i'm not sure how to interpret. One pertinent point near the beginning states : "World production of iron ore grew by 12% in 2006 to reach 1.5 billion mt (See Table 1). This was a fifth consecutive record high. " Vespine (talk) 04:28, 29 November 2010 (UTC)[reply]

How can a solvent be non-polar but be made of polar molecules?

My organic chemistry textbook makes the distinction between polar molecules and polar solvent. It says that polar molecules are identified by the high dipole (u) seperation of the molecule, while polar solvents are identified by a high dielectric constant.

It says that all polar solvents are made from polar molecules, but the opposite is not true and provides the example of formic acid vs acetic acid to demonstrate this.

Both formic acid and acetic acid are polar molecules by virtue of their dipole moment. However, formic acid, with a dielectric constant of 59, is also a polar solvent, while acetic acid, with a dielectric constant of 6.1, is not a polar solvent.

Why is this? Would both solvents be adequate for dissolving ionic compounds such as NaCl? Acceptable (talk) 08:05, 26 November 2010 (UTC)[reply]

Solubility of polar substances (and by extension, ionic) substances is dependant almost solely on dielectric constant. Your textbook is pretty much spot-on on this one. The solubility of something like NaCl is dependent on the ability of the solvent to solvate the ions; that is to make bonds to the ions which are stronger than the bonds the ions would make to each other (strictly speaking, it is defined thermodynamically; the substance is soluble if the free energy released in the formation of the solvent-ion bonds is greater than the free energy required to break the ion-ion and solvent-solvent bonds). Dielectric constant takes this ability into account, whereas dipole moment does not. --Jayron32 16:32, 26 November 2010 (UTC)[reply]
But why does formic acid have a higher dielectric constant than acetic acid when both have a carboxylic acid functional group? They differ only by the fact that one has a methyl group and the other has a hydrogen. Acceptable (talk) 21:24, 26 November 2010 (UTC)[reply]
The methyl group is MUCH more "electron donating" than the hydrogen atom is. You can think of this in two ways; either you can think of it as the methyl group donating electrons to the carbon part of the COOH dipole OR you can think of the methyl group as acting like a "positive charge sink", again to the same effect. The dipole moment is calculated for the bond, while the dielectric constant is calculated across the whole molecule; while the dipole moments will be similar (but not the same, due to the methyl's effect described above), the dielectric constant will be much lower on the acetic rather than the formic acid. You can see the effect even greater on very large molecules; take something like stearic acid; the dipole moment on the COOH will not be that much smaller than it will be on the acetic acid, indeed after propionic acid, longer chains of carbons do not markedly affect the dipole moment in that part of the molecule. However, the dielectric constant continues to go down to nearly nil. I am pretty sure that after 4 or 5 carbons, the bulk substance is considered essentially non-polar; despite the acid group. --Jayron32 21:33, 26 November 2010 (UTC)[reply]
Ah, ok, thanks a lot for the helpful explanation. Acceptable (talk) 02:08, 27 November 2010 (UTC)[reply]

Is it possible to vomit in your sleep and die by suffocation without being under the influence of anything?

In other words, could a perfectly normal, sober person who hasn't had anything to drink//taken any mind-altering substances vomit and thereby asphyxiate during slumber, or would the body's involuntary control measures trigger countermeasures like gagging, rolling over to one side, etc. before the person awoke?

Or perhaps it's not even possible to vomit in one's sleep? Sign me "curious" The Masked Booby (talk) 09:21, 26 November 2010 (UTC)[reply]

Did a few google searches on it, most of the results appear to be people asking about it on various forums - One answer on this forum referred to GERD - Although perhaps not vomiting in the sense that you mean, perhaps mildly along the same lines. All other mentions that I've come across thus far (the last few minutes), seem to point to factors enducing the vomiting while asleep: alcohol; obesity; underlying illness; directly related illness.. etc. Darigan (talk) 09:54, 26 November 2010 (UTC)[reply]
I think a better question is can you do that and stay asleep, and I doubt it. If I vomited I'd wake up for sure. Same goes x1000 for not being able to breath. Ariel. (talk) 11:15, 26 November 2010 (UTC)[reply]
I think the best anyone could say is it is unlikely. The human body is a fantastically unpredictable thing, so I would not be surprised to find, if I looked hard enough, isolated reports of one or two people who have died by choking on their own vomit, in their sleep, without any complicating factors. --Jayron32 15:22, 26 November 2010 (UTC)[reply]
One thing of course if we are thinking there is no illness or other factor as someof the above answers are discussing, vomitting itself is not common asleep or not. Nil Einne (talk) 16:50, 26 November 2010 (UTC)[reply]
I recall a report or two of people suffocating to death from their vomit while very drunk and thus probably unconscious. 92.24.178.149 (talk) 22:05, 26 November 2010 (UTC)[reply]

The suffocation reflex from carbon dioxide buildup in the lungs is very painful if you aren't used to it, and very powerful. Vomit is a fluid which is almost always easy for a conscious person to expel. A conscious person in shock may be effectively paralyzed, however, which is why CPR and related forms of first aid instruct the person administering the aid to check to see that the airway is clear, even when the victim is conscious. Ginger Conspiracy (talk) 01:03, 27 November 2010 (UTC)[reply]

Copper containing biomolecules

Why did copper containing biomolecules appear later in evolution than their iron based analogues that do the same job? —Preceding unsigned comment added by Blackmetalgrandad (talkcontribs) 10:57, 26 November 2010 (UTC)[reply]

I can't answer your question, but you should know that there is something like 100,000 times as much iron on earth as there is copper. Ariel. (talk) 11:22, 26 November 2010 (UTC)[reply]
Oops, I meant why did the copper based ones appear earlier!144.32.126.11 (talk) 11:33, 26 November 2010 (UTC)[reply]
It could be that hemocyanin would have been more efficient in the early seas which where oxygen poor, than iron based oxygen carriers. Also, it allows (or favours) simpler body organs. --Aspro (talk) 16:44, 26 November 2010 (UTC)[reply]

What is its appearance? Colorless gas? (just a guess) --Chemicalinterest (talk) 15:02, 26 November 2010 (UTC)[reply]

Yes, good guess! It sublimes at 4.8 °C under atmospheric pressure. Physchim62 (talk) 15:12, 26 November 2010 (UTC)[reply]

Iodine from caliche

The article does not state how iodine is extracted from the iodide and iodate in caliche. How is it extracted? All of these iodine questions are because of this. --Chemicalinterest (talk) 15:25, 26 November 2010 (UTC)[reply]

It can be reduced with sodium bisulfite but there are more advanced multi-step processes which seem to be more popular these days.[4] Ginger Conspiracy (talk) 00:54, 27 November 2010 (UTC)[reply]

Why kilogram as a base unit rather than gram?

Why was the kilogram chosen as a base unit of the International System of Units rather than the gram? The kg is the only base unit with an SI prefix as part of its name. Our article on the Kilogram says, "Since trade and commerce typically involve items significantly more massive than one gram...[the standard became] one thousand times more massive than the gram—the kilogram." Although this seems common sense, wouldn't the desire for consistancy in the new system be more important? Doesn't having one base unit that already contains a prefix in its name cause confusion when multiplying it by a another prefix? It seems that the term "kilokilogram" would be necessary to describe a mass of 1000 kg, which is awkward. --Thomprod (talk) 15:26, 26 November 2010 (UTC)[reply]

(1) "Since trade and commerce typically involve items significantly more massive than one gram...[the standard became] one thousand times more massive than the gram—the kilogram."
(2) No.
(3) No.
--Shantavira|feed me 15:45, 26 November 2010 (UTC)[reply]
The OP is making a common mistake; the confusion between the metric system and SI. The SI is a subset of the metric system chosen for convenience in the widest possible applications. The other thing about SI is that its "base" units are used to derive the so-called "derived units"; thus the Newton and the Joule and the Pascal and other units are always expressed as ratios or products of things like kilograms, meters, seconds, etc. There are other systems besides the SI which use different metric units, see cgs system, which uses units like the erg. In summation: The SI is not the metric system. It is a set of units speficially chosen from the metric system, chosen to be convenient for use in certain applications. --Jayron32 15:52, 26 November 2010 (UTC)[reply]
See grave (unit). Basically a grave (from gravity) was going to be the base unit; a gram was an alias for a milligrave; just as a ton is an alias for 1000 kg. However, this was just before the French Revolution, and Grave is also a French title; similar to the German Graf, or English Count. Grave as a title has the same etymology as Graff, which is different from Gravity. After the revolution it was felt that this would be contrary to égalité so grave as a unit was dropped. BTW, this came up a few months ago. CS Miller (talk) 16:00, 26 November 2010 (UTC)[reply]
Jayron: So the kilogram was chosen over the gram as the base unit of mass in the SI because the former was (and is) more "convenient" in most applications. I understand that. But didn't the kg stick out to the designers of the SI as the only base unit name to include a prefix? If I was designing a new system (based on mathematics and powers of ten and such) with base units and modifying prefixes (somewhat analogous to nouns and adjectives in grammar), why would I build in a point for possible later confusion by including a modifier in the name of a base unit? There is no color named "lightred", for example.
CS: I understand the politics of not using the word "grave". But, why didn't they just come up with a completely different word to represent 1000 grams, if that amount was thought to be more convenient than one gram? --Thomprod (talk) 17:00, 26 November 2010 (UTC)[reply]
Pass. Perhaps it was just an interim decision use gramme instead of milligrave, and no-one got around to making a new word for grave/killogramme. I made a slight mistake in my previous statement - originally the definition was the gramme - 1cc of water at 0°C, and the grave was the practical physical object to represent it. I don't know why the definition was moved to kg/grave - perhaps it was to make the definition and representation the same. CS Miller (talk) 17:28, 26 November 2010 (UTC)[reply]
Ok, now that makes sense. If the unit called "grave" was practical, but now politically incorrect, it would have made more sense to call it something brand new, rather than the hybrid "kilogram". --Thomprod (talk) 18:35, 26 November 2010 (UTC)[reply]
Don't expect SI to be this perfect, logical system. It's a system of units like any other. Use it when it's convenient; ignore it when its pronouncements are silly. In particular, ignore completely any recommendation not to use convenient units like the curie, or to avoid the name micron. --Trovatore (talk) 08:20, 27 November 2010 (UTC)[reply]

Force as Gradient of potential

We can define the quantity U such that dU is to equal -dr. But dU = ∇Udr. So F⋅dr = -∇Udr For this to be true in general, F = -∇U.

If a force can be written as the gradient of a scalar field, then this is taken as a definition that F is conservative. But where in the above derivation was F assumed to be conservative? 70.52.44.192 (talk) 20:54, 26 November 2010 (UTC)[reply]

Please do your own homework.
Welcome to the Wikipedia Reference Desk. Your question appears to be a homework question. I apologize if this is a misinterpretation, but it is our aim here not to do people's homework for them, but to merely aid them in doing it themselves. Letting someone else do your homework does not help you learn nearly as much as doing it yourself. Please attempt to solve the problem or answer the question yourself first. If you need help with a specific part of your homework, feel free to tell us where you are stuck and ask for help. If you need help grasping the concept of a problem, by all means let us know.
Having said that, you might benefit from reviewing force, gradient, and reading Newton's laws of motion#Variable-mass systems carefully. Ginger Conspiracy (talk) 23:52, 26 November 2010 (UTC)[reply]
No, it isn't a homework question, although looking back it does seem that way. The reason I spelled out the F = -∇U derivation was so that someone could point out more clearly where the assumption that F is conservative was made.
I looked at the articles you mentioned. I don't see what variable-mass systems have to do with conservative forces, and the gradient article doesn't mention potential energy. The force article starts by saying "a potential scalar field is defined as that field whose gradient is equal and opposite to the force produced at every point". But why can't such a potential be defined for any force? You might say, well U isn't a function of just r for all forces, just conservative ones. But if dU is defined as above, then it does seem, incorrectly, that all forces can be constructed to be the gradient of a scalar field. 70.52.44.192 (talk) 01:46, 27 November 2010 (UTC)[reply]
Oh, so by conservative you mean subject to conservation laws instead of covering only the particle case instead of the variable mass case? In that case, would the assumption of elastic force transfer be the reason, as opposed to force from an inelastic collision? Ginger Conspiracy (talk) 02:32, 27 November 2010 (UTC)[reply]
(ec) Such a U does always exist for a radial force whose magnitude is a function of only the radius, i.e., all such forces are conservative. But it doesn't always exist for more general force fields. For example, take . -- BenRG (talk) 02:37, 27 November 2010 (UTC)[reply]
F is assumed to be conservative in the line "We can define the quantity U such that dU is to equal -dr." If F is not conservative, there is no field U that satisfies this definition. Looie496 (talk) 05:35, 27 November 2010 (UTC)[reply]
In BenRG's example, wouldn't dU = -ydx + xdy satisfy dU = -F⋅dr? 70.52.44.192 (talk) 07:23, 27 November 2010 (UTC)[reply]
Yes, but what U satisfies dU = -ydx + xdy? -- BenRG (talk) 08:47, 27 November 2010 (UTC)[reply]
A conservative force is a force for which the work done by the force on a particle that moves from A to B depends only on the start and end points A and B and not on the path taken between them. Equivalently, a conservative force is a force for which the work done by the force on a particle travelling round any closed path is always 0. As has been said, a force that is the gradient of a scalar field is conservative - you can see this by looking at the integral of dr around any closed path. Also, any conservative force can be expressed as the gradient of a scalar field U by picking a fixed point A and defining U at any point B to be the integral of dr along any path from A to B - the conservative property means that this definition gives a well defined value for U at each point. U depends only on the choice of the point A, which is the same as saying U is uniquely defined up to an arbitrary additive constant.Gandalf61 (talk) 09:32, 27 November 2010 (UTC)[reply]

Thanks 70.52.44.192 (talk) 16:06, 27 November 2010 (UTC)[reply]

November 27

Absolute rest point

According to the book "Four Symphonies of an Oriental Universe" the author, Zhi-Zhong Cai, said we can locate an absolute rest point with some imagination. Let a point-source of light shine once at a point E, then the out going wave front will make a growing but thin out ball of light. Do you agree that the center of that growing ball of light is a fixed point as in the absolute rest universe? The reason is clear, because the speed of light is independent of the speed of the source of light so that the growing ball of light is growing from a fixed point in the whole universe or whole space. The only difficulty is that, unless the source of light stays shining like once every two seconds at that same event point E while the ball of light is growing, otherwise, we have no way to point out where is E. Because our technology is unable to show us the actual growing ball yet. It is the same for every event with a point-source of ligh shines once so that the location of that event is an absolute rest point that is secured by the growing ball of light, if we can see it, then we can try to locate its center point. That means, if we shine four point-source of lights at each of (0,0,0),(1,0,0),(0,1,0), and (0,0,1) of a Cartesian coordinate system roughly at the same time then we have an absolute rest frame. We cannot deny their existence just because we cannot figure out some way to use those growing balls of light to locate their centers. Please comment, thanks.Jh17710 (talk) 06:13, 27 November 2010 (UTC)[reply]

I don't understand why that reference frame would be anymore 'absolute' than any other reference frame or why it would be considered an 'absolute rest point' or even how it would be determined to be 'at rest' unless you just wanted to define it that way in your coordinate system. This may help. Sean.hoyland - talk 07:23, 27 November 2010 (UTC)[reply]
Thanks for the reference. The reason why it is an absolute rest point to any absolute rest frame is based on the character of light. Since the way light moves away from the souce point is independent from the speed of the source. That means, if just shine once, the center point of that growing ball of light is guarded by that thinning out and growing ball of light as an absolute rest point in the whole space; at least at time while the ball is recognizable. The problem is that one single growing ball of light will thin out as 1/(r^2) of density and the speed of light is too fast that scientists have no way to watch that growing ball so that even the growing ball can show them an absolute rest point, its center point, they have no way to mark it yet. But, there are rest points if we can figure out some way to see those growing balls to locate their center points.Jh17710 (talk) 22:30, 27 November 2010 (UTC)[reply]
Actually, all reference frames will agree that the expanding ball of light is spherical, and that its center is at rest. So you can't use it to pick out a preferred reference frame — all the frames will think they're the preferred one. --Trovatore (talk) 07:25, 27 November 2010 (UTC)[reply]
Trovatore, thanks for yuor kindness to show me the right way to describe that expanding ball. Let's look at the issue of spherical shap first. Because of the farther length looks shorter, the only perfect circle shape of a ball is when the ball is infront of us or when we turn our eye sight directly to it. That means even within the same reference frame, the spherical shap is not automatically maintained. Secondly, not all inertial frames can think they are the rest one and it will be very easy to figure out if, well, the if part is very difficult; if we can locate the expanding ball, then, instantly locate the center point, then, instantly decide if that center point is at REST in their systems.Jh17710 (talk) 22:30, 27 November 2010 (UTC)[reply]
OK, first of all, we need to distinguish between measurement in a reference frame, and visual appearance in a frame. There are complications involved in the visual appearance of objects at relativistic speeds (we surely have an article on that, but I'm not sure where it is).
But those complications don't interest us here. What we're concerned about is measurement of time/space coordinates. For example, you might set up a spherical shell of photodectors, all with synchronized clocks. When they detect the light, they send out a signal, time-stamped by what their clocks say. If all of those time stamps are the same, you conclude that the shell of light is centered at the center of the photodetectors.
With me so far? Here's the surprising thing: No matter what reference frame you happen to be in, you can set up a concentric system of such shells, according to which you will conclude that the center of the shell of light is at rest. The why of that is a little involved to explain in detail, but it all works out. --Trovatore (talk) 22:45, 27 November 2010 (UTC)[reply]
Trovatore, I mean, it is really hard to figure out if a shape is spherical at far away, because our tools are very limited; it is even more difficult to measure from another inertial system, no tool at all. But you are right, that is not the main point. On the shell of photodetectors, we will need four wave peak counters at tips of the largerest internal triangular pyramid, let the center send out fixed frequency signals, then, if all four frequencies remained the same for a little while, then, yes we are lucky to witness an absolute rest point at that little while. I don't trust clock.Jh17710 (talk) 00:40, 28 November 2010 (UTC)[reply]
I made a wrong statement above. I will say it at the end of this paragraph. I found out it is not easy to synchronize clocks. If we think a little bit deeper, the four clocks on the spherical shell cannot have the same gravity so that their speeds will be different! Even if we can adjust their speeds, the mistake I made is that, all wave peak counters will count the same frequency within one minute of time, since we assumed that their distances from the source of light are all fixed. The first wave front may reach four clocks at different time, but, since all the following waves are all have the same differences in between four clocks so that the wave peaks count at each of four clocks will be the same in the same period of time. Sorry for the mistake.Jh17710 (talk) 15:43, 28 November 2010 (UTC)[reply]
The problem of detecting the wave front of that SINGLE expanding ball of light is our technology is way behind on two issues relative to measurement, one is to adjust the speed of four clocks so that they are the same against to gravity and temparature or other factors, and another one is to match the numbers showing on four clocks. Until we can control the synchronization of four clocks in this discussion, we have no way to find out if the SINGLE expanding ball of light reaches four clocks at exactly the same time.Jh17710 (talk) 15:43, 28 November 2010 (UTC)[reply]
Well, there is no "exactly the same time" (or "exactly" anything) in engineering. But you don't need it. You put the detectors in free space. They keep their relative positions constant, and their clocks synchronized, by sending laser messages to each other with timestamps, and making corrections with little hydrazine rockets when necessary. It's not going to be perfect, but it can narrow down the center of the expanding ball of light within some reasonably small margin of error.
And you can put several larger collections of these assemblies of photodetectors at larger distances, and decide whether the center of the ball of light is moving (again, within some margin of error) in your reference frame.
Then the surprising, counterintuitive thing is this: No matter what reference frame you are in, you will determine that the center of the ball of light is at rest, within your experimental error. --Trovatore (talk) 20:59, 28 November 2010 (UTC)[reply]
Trovatore, there are quite a few issues in your last comment. First of all, if you like to let four detectors keep constant relative positions do you think it may be better if we use one solid frame to install them? The second issue is laser messages with timestamps, since they are at four tips of a triangular pyramid, that can be a good idea to synchronize their clocks; but, the top one with different gravity potiential may adjust its speed a little bit to match other 3 clocks at about the same altitude. Then the third issue is if we have two sets of this kind of shell, one is rest and the other one is moving directly away from the rest set at any speed then back and forth at any other speed, we should find out, logically speaking, all of eight devices will count the same total amount of wave peaks within same period of time. Do you expect this result? The fourth issue is that above result does not mean both center points are absolute rest points. The key point is the SINGLE expanding ball of light, does it reach four devices at the same time? Or, we can arrange the experiment in one dimensional space to get more chance of being rest and to make things easier, just use a section of train. Logically speaking, if we fire two bullets at the same time at the center of that section of train to two ends, then, no matter how that train moves, bullets reach ends at the same time, but, not if we fire two rays of light. Is that right?Jh17710 (talk) 02:22, 29 November 2010 (UTC)[reply]
Let me just take your last question, because it's the one that treats the key issue.
No, in fact, that's not right. If you fire two rays of light forward and backward from the center of a train moving at a constant speed, they will (as measured in the train's frame) reach the ends of the train at the same time, no matter how fast the train is moving. --Trovatore (talk) 06:27, 29 November 2010 (UTC)[reply]
That is the key issue. Looks like your main point is on the "measured in the moving system". That means, if measured in the rest system and if when v=o, both lights hit ends at the same time then when v>0, the light will hit the rear end first. Is that what you mean?Jh17710 (talk) 03:02, 30 November 2010 (UTC)[reply]
No, it isn't. I meant precisely what I said. If it's hard to make it square with your intuition, that's fine; that's because your intuition is wrong. You need to get a new intuition, one that's right. The way to do that is to work through why what I said is right. --Trovatore (talk) 03:12, 30 November 2010 (UTC)[reply]
Oops, sorry. I didn't see your "measured in the rest system". Yes, you're right. --Trovatore (talk) 03:49, 30 November 2010 (UTC)[reply]

Thanks. Now I have to study about this issue for a while.Jh17710 (talk) 04:14, 30 November 2010 (UTC)[reply]

Simultaneity

The problem boils down to relativity of simultaneity. The center of a growing light ball is the spatial center of a set of events which occur at the same time. But observers disagree about what "at the same time" means, which makes them disagree as to where the "center" of the growing light ball is. Red Act (talk) 08:21, 27 November 2010 (UTC)[reply]
Red Act, yes, the center of an expanding ball of light is the spatial center of a set of events, like when the radius of that ball is at 10', 20', 30',...but I don't see you point of "at the same time". Could you explain it a little bit? Thanks.Jh17710 (talk) 22:30, 27 November 2010 (UTC)[reply]
I assume you're familiar with conic sections. If you slice a cone along parallel planes at the correct angle, you get circles; the centers of those circles lie on a line through the cone. If you slice the cone along parallel planes that are inclined to those other planes by less than 45°, you get ellipses. The centers of the ellipses (for any particular set of parallel planes) also lie on a line, but it's a different line. The slope of that line is the velocity of the inertial reference frame whose planes of simultaneity are the parallel planes that you used. Thus, depending on which reference frame you use, you get a different "center". That center is always at rest with respect to the reference frame that you used. -- BenRG (talk) 01:14, 28 November 2010 (UTC)[reply]
BenRG, for events happen at points, lines, or even plans, we may have problem of "at the same time" or different time, but, this will not happen for an expanding BALL like this case. They will never happen at the same time from any observers. Am I correct?Jh17710 (talk) 03:25, 28 November 2010 (UTC)[reply]

Absolute time

According to the book "Four Symphonies of an Oriental Universe" the author, Zhi-Zhong Cai, said we can have a better clock than current atomic clock. If I have some device can count wave peaks then just select a source of light with fixed frequency which is moving at constant speed v relative to me along the line connecting that source and me, and we are ready to use that device to measure time. According to Doppler effect, no matter the moving party is the source or me and no matter what is the value of v, the frequency and the wave length of light may change but the speed of light will remain the same, so that my time can be calculated by (the total wave peaks I count by the device x the wave length)/(the speed of light). That means, the time period calculated at different relative speed will be the same. Could we use this idea to make a clock to measure time period? If yes, then, a moving clock will not slow down. If the device can be made very small, then we can put the device and the source of light in a small vacuum box and use it as a clock, that will be v=0, and since the speed of light is independent of the change of gravity, it will be a better clock than atomic clocks. Isn't it?Jh17710 (talk) 07:01, 27 November 2010 (UTC)[reply]

Yes, you can make a clock that measures a time coordinate independently of your motion if you use an external reference. A more familiar example is a cell/mobile phone that gets its clock setting from the cell tower. Another theoretical example would be a clock that measures the temperature of the cosmic microwave background, though that's cooling so slowly that it wouldn't give you a very precise time in practice. There are also inertial navigation systems that, in certain restricted cases, can keep time independent of your motion without any external reference. But this has nothing to do with the accuracy of atomic clocks. They do a very good job of what they are designed to do, which is count very nearly equal intervals of proper time. The clocks I described before are measuring some form of coordinate time, which can also be useful, but it's not the same thing.
I'm not familiar with the book you mention, but I would not use it as a way to learn physics. It sounds like the author is pretty clueless. A lot of people like that write "physics" books, for some reason. -- BenRG (talk) 09:43, 27 November 2010 (UTC)[reply]
Thanks. That book just published 9-1-10 at Beijing, it is in Chinese.Jh17710 (talk) 03:26, 28 November 2010 (UTC)[reply]
The article Comoving_distance#Comoving_coordinates covers comoving time in a bit of detail. --Jayron32 13:48, 27 November 2010 (UTC)[reply]
I think there are at least two issues. First, the speed of light does change in a gravitational field. Second, and more importantly, the method depends on the distance between the two objects remaining constant to an extremely high degree of precision. This means that any vibration, such as caused by sound waves, will cause a wobble in the distances. I don't see how it would be possible to solve that problem. Looie496 (talk) 17:05, 27 November 2010 (UTC)[reply]
Looie496, the frist issue is still unknown, we don't even know if the gravity will change the direction of light; the change might be caused by the different density of air around the sun. The second issue is very true. It is even worse for a constant velocity v in between two objects. However, the spirit in the idea is that, theoretically speaking, in front of a light source, we let a counting device move under a relative constant velocity v, then, if we use (total wave peaks x the wave length)/(the speed of light) to define time, that kind of time will be independent of v. What do you think?Jh17710 (talk) 23:13, 27 November 2010 (UTC)[reply]
The GR prediction of gravitational lensing has been experimentally confirmed to very high precision. Maybe 90 years ago you could have argued that it was refraction in the Sun's atmosphere, but not today. See Tests of general relativity.
The questions you ask in this thread about time in special relativity could be rephrased as questions about position in Newtonian physics. Can you build a device that determines its absolute position (relative to Earth's surface) by triangulating with cell towers? Yes. Can you put the cell towers and the receiver together in a box to get a device that knows its absolute position without an external reference? No. Can you use inertial navigation to keep track of your absolute position without an external reference? Yes (subject to certain caveats). In special relativity the same things are true of the time coordinate, for essentially the same reasons. -- BenRG (talk) 00:36, 28 November 2010 (UTC)[reply]
That way of counting time is just an idea. Like Looie496 said, it is almost impossible to make a device to count wave peaks due to the difficulty of keeping same distance. However, even if gravity can change the direction of light, the speed is the same so that the idea of that book still works, right? About a device to locate an absolute rest point, it is even more difficult as discussed in the thread above this one.Jh17710 (talk) 01:58, 28 November 2010 (UTC)[reply]
Pulsars are also better "clocks" than nuclear decay. ~AH1(TCU) 03:27, 28 November 2010 (UTC)[reply]

If we have source and device

Even if you had such a light source, and an object capable of counting the light pulses, that still won't tell you how much time passed. You'd have to know the frequency of the light being emitted. You'd need another clock to figure out how fast that one goes, at which point you might as well use the other clock. — DanielLC 05:52, 28 November 2010 (UTC)[reply]
DanielLC, the key issue of a light source is having stable frequency. We have some known natural frequencies and radar will emit EM waves with almost fixed frequencies. So long as the souce and the device are keeping same distance or one of them is moving away directly from each other at constant speed (I will say this is impossible in real world, but, theoretically we should include the relative moving situation.) then the device will receive EM waves with fixed frequence. If we do have a known source, like red light, and a device, then we don't need any clock. That source and device make a perfect clock themselves so long as we can find some way to keep a constant distance between them. With the known frequency, we can calculate the time by the total count of wave peaks. Right?Jh17710 (talk) 17:22, 28 November 2010 (UTC)[reply]

Force at a distance without any connection

How does the force from gravity or magnetism communicate itself to distant objects, when nothing connects or passes between them and the attractor? And there's no aether either.

I'm aware of the teaching model that looks like ball-bearings on a rubber drum, but what is this curved surface and the third-object gravity embodied in in reality? 92.15.11.45 (talk) 14:18, 27 November 2010 (UTC)[reply]

There isn't any need for solid objects to be involved. In fact, what we see as solid objects ... aren't. The electromagnetic repulsion between electrons is what keeps the things we see as solid about an angstrom away from each other instead of passing right through each other. It's more like the whole universe is made of infinitely-small particles than of things that can actually touch each other. Paul (Stansifer) 15:02, 27 November 2010 (UTC)[reply]
I'm already aware that things are made of atoms and smaller things. That does not explain how for example a star pulls on the planets orbiting it, or how a magnet attracts things from a distance. 92.15.11.45 (talk) 15:28, 27 November 2010 (UTC)[reply]
Just about everything attracts or repels anything else at a distance. --Chemicalinterest (talk) 15:37, 27 November 2010 (UTC)[reply]
I'm asking how this happens. 92.15.11.45 (talk) 15:45, 27 November 2010 (UTC)[reply]
When I first read your question I found the article Action at a distance (physics). This didn't seem much help and I didn't understand it, so I didn't mention it, but on reflection, maybe it's helpful to you, so there you are. There seems to be some debate about how it happens and what it means. 81.131.46.126 (talk) 16:32, 27 November 2010 (UTC)[reply]
This is a good question, and has been answered on the desk many times. Action at a distance was one of the most plaguing problems of 20th century physics, but for the most part it has been resolved. There are a few critical mechanisms that apply to different kinds of fundamental interactions:
  • Forces like electromagnetism and the strong nuclear force are mediated by a particle, the photon or gluon. These forces "propagate" at the speed of their carrier-particle, and in the case of the photon, that particle also conveys energy and momentum. The mathematics to ensure energy-conservation are explained by treating force-carriers as virtual particles until they undergo an interaction.
  • The special case of gravity is explained by general relativity - there is currently no known force-carrying particle for gravity. Instead, Einstein's theory of gravity explains that all mass perturbs the geometry of space - so simply by existing, massive objects cause other objects to change their trajectories. Gravity therefore changes the shape of the inertial trajectory by warping the geodesic of space. General relativity mathematically describes the relationship between mass and the resulting warping of space-time geometry - in other words, allowing you to calculate the effective force. It also can describe how quickly gravitational information propagates - that is, when you move a massive object, how soon other objects in the "neighborhood" realize that something has changed.
  • The last case, quantum entanglement, applies to certain quantum properties, but in general does not convey energy or information.
Hopefully this explains the various ways things can "act at a distance." Nimur (talk) 16:35, 27 November 2010 (UTC)[reply]
Note that (virtual) gravitons are hypothesized as the force carrier particles for gravity. And if you're having a hard time understanding how gravity can both be transferred by a particle and by curvature in space-time, you're not alone. Reconciling the two is one of the key stumbling blocks in constructing a theory of everything which combines quantum mechanics (forces-as-virtual-particles) and general relativity. -- 174.24.198.158 (talk) 19:10, 27 November 2010 (UTC)[reply]

you might be intested in this answer. 62.54.13.205 (talk) 23:04, 27 November 2010 (UTC)[reply]

Midnight sun question

Because of possible changes to the earth's tilt on its axis and to the earth's orbital path around the sun, is it possible for a place right around or near the Arctic Circle to barely experience the midnight sun on one year during the summer solstice night, but on another year, not experience the midnight sun let's say, for 10 minutes during the summer solstice night? Willminator (talk) 15:43, 27 November 2010 (UTC)[reply]

Well, the largest nutation is only some 20 sec of arc. On the Earth's surface that's about 2000ft of distance. The sun takes only 2 minutes to cover its own diameter. So on a very flat piece of land at approx 66½ degrees north (or south, six months later) you might just be able to catch a 'momentary' extension (or reduction) in night-time, as measured against the previous year. The sun would cover the 2000 ft in about a second or so I would think. This is a bit rough and ready, so I think it needs someone else to double check the assumptions I'm making.--Aspro (talk) 16:47, 27 November 2010 (UTC)[reply]
I don't see where you get the 2000 ft from (20 seconds of arc north/south on the Earth would be 1215 ft)(no, you were right about the distance), and while the Sun does indeed cover that apparent angle in less than a second, it's not what matters here, since the direction of the Sun's apparent motion is very close to east/west in the case of the midnight sun. What cannot happen is that the Sun never touches the horizon in one year but sets completely below the horizon in the next year (or the year after that), because the Sun's apparent size is roughly 30 minutes of arc. And if you consider not nutation but the fact that days and years don't align (i.e. if the exact time of solstice is at midnight for one location, it will be roughly at noon for that same location 2 years later, since 1 year is approximately 365 and a quarter days), then the difference will be only 3.3 seconds of arc (the formula is arccos(sin(axial tilt)*cos(angle that Earth travels in its orbit in half a day)) - arccos(sin(axial tilt))). Icek (talk) 23:43, 27 November 2010 (UTC)[reply]
Good questions. To do it in my head, I assumed a nautical mile is about 6080 ft and the illuminated parabola as a simple triangle with a 'lateral' base (therefore) of about 2000 ft west to east (not east-to-west as you put it because that is not the way the sun orbits), and about 2000 ft in the NS axis to keep things simple. I wanted to avoid discussions about twilight because it would be obvious to all that the effect would not be great enough to go into proper night. Likewise, whether 'day' starts at first rays or at the full disc becoming visible, is more of a academic question than what the OP is really asking. So I am assuming sun's rays. Perhaps the OP can comment on whether s/he meant this. The point about 'two years' is misleading pedantism in regards to the spirit behind the question. For we are talking about the apparent altitude extreme at the solstice, not a date on somebodies calendar. For it to be noticeable to the eye, the ground would need to be very flat, or you would need to assume the exact same position each year. Yet, if one used say a sextant, the difference in altitude -I expect - would be just discernible. Be on guard though. In such cold climes atmospheric 'tunnelling' may cause the sun to be visible when it is actually below the visible horizon. --Aspro (talk) 01:05, 28 November 2010 (UTC)[reply]
By midnight sun I mean the sun's disk being above the horizon at night in the summer solstice. A place that gets white nights is where the sun's rays are visible in the middle of the night, but the sun stays below the horizon around summer solstice. What I'm talking about is about a place where at least half of the sun's disk is above the horizon right in the summer solstice night, which happens around the Arctic Circle line. What I'm asking is if there a year around the Arctic Circle line where at least part of the sun's disk is exposed in the middle of the summer solstic night, but on another on another year in the summer solstice night it's not, or will the sun's disk be always partly exposed at that such location every year on the summer solstice night. I hope I've clarified what I tried to ask. Willminator (talk) 03:04, 28 November 2010 (UTC)[reply]
In that case the answer is no. The sun's angular diameter is about 1920 arcseconds and we are only talking about a change of about 20 arcseconds in altitude. As this would only represent the edge of the disc showing or not showing, it would be unlikely to cast a shadow bright enough to be seen above the twilight. However, with the aid of a post driven deep enough into the ground to prevent frost movements' so as to provide a fixed sight-line to the horizon, it should be able to indicate that in some years a few rays of midnight sun are still observable and in other years there is a brief moment of no disc to be seen at all. The effect though is very small and not something one would notice unless one took the trouble to observe it carefully. Even then, as I have already mentioned, the closer to the horizon that one observes events, the more the atmosphere can distort the true position of an object and make those observations unreliable. So the effect is of academic interest only.--Aspro (talk) 11:19, 28 November 2010 (UTC)[reply]
If you want to take a long-term perspective, the Earth's obliquity periodically changes from 22.1 to 24.5 degrees with a ~40,000 timescale due to interactions with other planets. So, if a hypothetical observer that could stay around 20000 years, then they would notice a quite substantial variation in the range of the midnight sun, even though these changes are pretty negligible on the scale of human lifetimes. Dragons flight (talk) 11:43, 28 November 2010 (UTC)[reply]

Aspro, which illuminated parabola are you referring to?
Why do you think that my 2 years have anything to do with the different human-made calendars? All I wanted to say is that the exact time of the solstice - the point in time when the angle between Earth's axis and a line connecting Earth and Sun reaches a minimum - may be at midnight for one particular location and one particular year. In the following year the exact time of solstice will be shifted by about 6 hours in local time of day, and in the year after that by about 12 hours. Obviously the distance in time to the next midnight cannot be more than 12 hours. So for a midnight 12 hours distant in time from the exact time of solstice the Sun's apparent position is 3.3 seconds of arc lower than for a midnight at the exact time of solstice (if there was no change in axial tilt). For 6 hours it would be 0.8 seconds of arc. Icek (talk) 11:55, 28 November 2010 (UTC)[reply]

The places you can see the midnight sun are those north of the polar circle. The arctic one is moving north by 14.2 metres per year. make of that what you like :-) EverGreg (talk) 20:48, 28 November 2010 (UTC)[reply]
Aspro, explain to me if I misunderstood anything in this discussion. If the partial midnight sun, or where part of the sun's disk is above the horizon at the sun's lowest point in the middle of a summer night, happens only once a year always every single year on the summer solstice night, which is possible on a location at or near one of the polar circles, like somewhere around Fort Yukon, Alaska; why do solar and lunar eclipes happen once in a while? It seems as if the earth's tilt axis and the earth's eliptical, orbital path around the sun change is not the same it was the previous year. Willminator (talk) 20:10, 29 November 2010 (UTC)[reply]
If I may answer as long as Aspro doesn't: Eclipses - solar or lunar - depend on the orbit of the Moon and are unrelated to what we were talking about. You could ask about the visibility of the Moon.
Having said that, there is a Moon-related effect on the apparent altitude of the Sun: Due to the motion of Earth around the common center of gravity with the Moon, Earth's center will be above and later below the ecliptic plane by a few hundred kilometers every lunar orbit. This results in a shift of the apparent position of the Sun by about 1.7 seconds of arc - also a very tiny amount. In fact, one often makes the assumption that the Sun's apparent position in the sky relative to the stars is the same everywhere on Earth, but there is indeed a variation larger than the one having to do with the Moon's orbit, about 18 seconds of arc from one side of the Earth to the other (e.g. between north pole and south pole on equinox; this may be called parallax). Icek (talk) 14:46, 30 November 2010 (UTC)[reply]

What does unranked mean in wikipedia's Scientific classification ?

What does unranked mean in wikipedia's Scientific classification ? In the horticulture class I am taking they dicuss Plant Hierarchical Classification. Kingdom,Phylum,Class,Order,Family, Genus, Species. There is no "unranked". Does unranked refer to the plant or the subdivision of the classification. Is wikipedia using scientific classification or something else? —Preceding unsigned comment added by Zoztrog (talkcontribs) 17:12, 27 November 2010 (UTC)[reply]

It is impossible to cleanly fit the Linnaean categories to the evolutionary trees produced by modern evolutionary biology -- there aren't enough levels, and the Linnaean levels don't have any basic underlying biological reality anyhow. "Unranked" means a grouping of species that does not match up with any of Linnaeus's levels. Looie496 (talk) 18:13, 27 November 2010 (UTC)[reply]
As a footnote, my experience is that textbooks tend to run about 10 years behind the leading edge of science. I expect that the textbook presentations will gradually change over the coming decade. In current biology, the Linnaean scheme is definitely on its way out. We can expect the concepts of genus and species to stick around, but above that, pretty much every Linnaean grouping is useless. Looie496 (talk) 18:19, 27 November 2010 (UTC)[reply]
Agreed. While I think that the lagging of textbooks behind the leading edge of science has its upsides (some call it the "bleeding edge" for a reason), what disturbs me is the way in which clearly-dated dogma is still being taught in classrooms. The Linnean hierarchy is still being drilled by rote in many universities, rather than using it as a shared construct with a tangible (if flawed) basis in observation. I hope that university biology faculty take the leading edge into account when they teach, and avoid having students memorize things that will have no utility in a few years. It's hard to teach concepts, but it's rewarding. -- Scray (talk) 21:06, 27 November 2010 (UTC)[reply]
If they're still teaching Linnean classifications, they're probably more than ten years past the leading edge then. That stuff was already well on the way out fifteen-plus years ago when I was at university and it's not like I was reading cutting edge bio stuff at the time. Matt Deres (talk) 14:41, 29 November 2010 (UTC)[reply]

CLONE

THE PROCESS OF CLONING,IS SEXUAL OR ASEXUAL REPRODUCTION —Preceding unsigned comment added by 41.211.232.70 (talk) 17:19, 27 November 2010 (UTC)[reply]

Our cloning and asexual reproduction articles should clear this up for you. Is this homework by any chance? -- Scray (talk) 17:57, 27 November 2010 (UTC)[reply]
See Dolly (sheep). (and please don't write in ALLCAPS) ~AH1(TCU) 03:22, 28 November 2010 (UTC)[reply]

Growth of trees

I've searched and maybe I just don't know how to use it well, but I'd like to know, If I make a notch/ scratch/ marking in a tree, will the marking rise as the tree grows? or will it stay the same distance from the ground? This is for research for a book I am writing. thank you!

46.117.94.124 (talk) 21:18, 27 November 2010 (UTC)[reply]

The only part of the tree trunk that is living or growing is the cambium. So the tree trunk just gets fatter with new rings, whilst only the new shoots grow upwards from the top. In other words, a blaze will stay at the hight that it was made. Does that help? --Aspro (talk) 21:33, 27 November 2010 (UTC)[reply]
A tree (and almost all plants) grows from the top, so a mark will stay at the same height. However grass is unusual - it grows from the bottom. Ariel. (talk) 23:09, 27 November 2010 (UTC)[reply]
You can also see this with signs, e.g. street signs, that are put on trees. The signs will remain at the same height above the ground. The tree may eventually even grow around the sign if it's there long enough. There is an example of this not far from the house I'm staying in now. The bark is slowly enveloping the signs. Dismas|(talk) 00:50, 28 November 2010 (UTC)[reply]
Pics or it didn't happen. -- BenRG (talk) 02:30, 29 November 2010 (UTC)[reply]
I was actually going to ask the same question! I recently moved into a house that has a data cable which is secured to a tree between the street and my house. The anchor is barely half way up the tree and I was wondering if I would have to have it moved at some time in the future. I did suspect the answers above but glad to have it confirmed. Vespine (talk) 23:15, 28 November 2010 (UTC)[reply]

November 28

Sleep and lifespan

A friend of mine said he wants to "experience a longer life", so he's going to cut out an hour of sleep from now on. I thought this was pretty bizarre logic. It got me thinking though, if you were to cut out an hour of sleep from now on, would that shorten your lifespan? Would the extra hours you stayed awake equal the additional hours you would have been alive for? ScienceApe (talk) 02:34, 28 November 2010 (UTC)[reply]

Sleep deprivation can lead to immune system and other health problems. However, if your friend is cutting from say 8 to 7 hours of sleep a day as an adult, this could potentially increase lifespan. Simply not sleeping, and yet still expecting one to live much longer is not possible. ~AH1(TCU) 03:18, 28 November 2010 (UTC)[reply]
You may be interested in scheduled napping to achieve more time awake. — DanielLC 05:41, 28 November 2010 (UTC)[reply]
I believe there have been studies in mice which show that limiting sleep has a positive effect on lifespan. Here is one article I found, i'm not sure how reputable the source is, Link. Calorie restriction has also been found to increase lifespan in mice but it's thought to not have a huge effect in humans, but there is an effect Link. But you have to ask yourself, if you're going to go through life tired and hungry, is it worth the few additional years at the end of your life? Vespine (talk) 23:13, 28 November 2010 (UTC)[reply]
And, of course, it bears stating that what applies to mice doesn't necessarily apply to humans: we are fairly different, particularly in this case. After all, sleeping and dreaming clearly has some important effects on our brains, and our brains are very different to mouse brains. And a lot of popularly-reported studies on how much sleep people need seem to give an average, which is reported as a universal optimum, when of course there is a lot of individual variation. I suspect this is more a problem of the reporting than the studies. Caveas. 86.161.109.130 (talk) 00:50, 29 November 2010 (UTC)[reply]

Looking for name of strange wart-like cutaneous growth on human foot with tentacles

Hi. First off, a few notes, I am NOT looking for medical diagnosis or opinion ... except that "diagnosis" might be a sliding rule here. I do not have this problem, no one I know has this problem, and I am not going to lie and say this is for a "book I'm writing". I have been looking every few months for this gross thing, and I can't find it, so I'm turning to this as my last resort. I'm merely curious what it is.

20+ years ago, I caught something on my foot, possibly from a public swimming area (I swam a lot). I went to the doctor and had it treated and it's never returned. But in my adult years, I have been curious as to what exactly it was, since I no longer have medical records of this thing. So please: I hope someone can tell me what it was.

It is not a plantar wart. But it was like a very large wart on the heel of the foot, and it had thick, white tentacular cilia growing from the center. It was really gross, I admit. To cure the thing, I had to pluck the tentacles every night and then apply a strong liquid.

One could ask why on earth I would want to remember such a thing, but I'm simply curious. Again, if I had this problem in real life, at the moment, I would immediately go to the doctor's. I just want to know what it was that I had. Anyway, if this question still breaks some rules, feel free to remove it. Thank you, Reflectionsinglass (talk) 09:25, 28 November 2010 (UTC)[reply]

Have you been through our slightly gruesome article List of cutaneous conditions? 87.81.230.195 (talk) 10:16, 28 November 2010 (UTC)[reply]
I have! But not thoroughly; it's such a long list, and it's one of the pages I'd been perusing. If it's listed there, it would take me quite a while to find it. I just took another gander, but no luck. Lots of other interesting things that prove to me I was not born to be a doctor :) Reflectionsinglass (talk) 10:53, 28 November 2010 (UTC)[reply]
To discount a parasitic infestation: What geographical location were you in when you first noticed it and for the few weeks before? Was blood or other liquid observable when tentacles were removed. Was it painful or just uncomfortable during the day and during tentacle removal. Did you think of the medical tincture applied as being 'strong' because it had a pungent acidic smell, or of alcohol or did it smart, etc.? Were the tentacles smooth all round or did they have texture? Did they end as a point or flattish or bluntly. Where they elastic or not when pulled ( i.e. did they become longer and thinner before coming out). --Aspro (talk) 11:37, 28 November 2010 (UTC)[reply]
Hi Aspro. At the time, I was in Connecticut, though I don't recall what season (in case that would be helpful). No blood, no other liquid that I recall. It was not painful, though I do remember some of the tentacles would hurt (if I tugged on them) and I'd leave them alone, but evidently I used tweezers to pull out the ones that didn't hurt. They were smooth; they were the color of cooked white rice and about the same size, with pointy ends, and yes, slightly elastic. I don't know what my mom did with them afterwards, probably just threw them away. I recall the smell of the medicine well: it was not acidic, it was more "mechanical" smelling, like a thick alcohol, definitely pungent. Reflectionsinglass (talk) 04:16, 29 November 2010 (UTC)[reply]
The "tentacles" could be hyphae, suggesting something like Phaeohyphomycosis. DuncanHill (talk) 14:39, 28 November 2010 (UTC)[reply]
Whoa! No, definitely not that, this wasn't any serious skin lesion like that. Just a bump on the foot with white things coming out of it that had to be plucked and medicated. Went away and I haven't had a problem since. I won't discount "hyphae" in general, but definitely not the ...mycosis. Reflectionsinglass (talk) 04:16, 29 November 2010 (UTC)[reply]
If there is a scientific answer to this, it would be a diagnosis that might still be relevant to the OP's health. To arrive at that diagnosis, we would need a back-and-forth exchange of information about symptoms, signs, other conditions. It seems clear to me that this is a request for medical advice, and I suggest that this should end here. -- Scray (talk) 17:03, 28 November 2010 (UTC)[reply]
Hi Scray. The people here, including you, know best when to stop. There were no symptoms, signs, or other conditions except what I mentioned above... this would be in the vein of asking, "What was it that I had that was black, hairy, and all over my tongue?" and the answer would be black hairy tongue syndrome (or whatever lol). Not looking for any medical advice, I'm not asking, how it should be treated. In fact, whatever it is that I had was obviously treatable, it was treated quickly, and I've never had a problem since! I honestly think I've given all the info I can, this was a very long time ago. I just thought maybe someone here would have a quick and definite answer, and it didn't hurt to ask. Anyway, one last go, if anyone has any ideas? Reflectionsinglass (talk) 04:16, 29 November 2010 (UTC)[reply]

The original poster wrote: "I went to the doctor and had it treated". If that doctor's still around, I'd suggest writing to them to ask what the thing was. You never know, they might still have records. --Anonymous, 03:22 UTC, November 29, 2010.

I doubt it; it's been over 20 years. And tbh, this is just a matter of curiosity. I really won't be hunting this information down. It was just such a curious thing that I caught. Anyways, thanks for everyone's help so far! Again if the matter is over-the-line, feel free to close discussion. I'll check back again soon. Reflectionsinglass (talk) 04:16, 29 November 2010 (UTC)[reply]
Thanks for the extra information Reflectionsinglass. Without venturing as far as giving a diagnoses, I think I can explain what happens in these cases. This sort of infestation is most frequently found in the tropics and sub-tropics and rather more rarely in temperate zones but they do happen. A fly will choose a human as its host on which to lay its eggs -especially if one has fallen asleep whilst fishing. A very effective and simple way of dealing with this event is to force the lava out by restricting their air supply with an oily or greasy substance. If this also contains a little turpentine or something similar, that will help keep the wound sterile as well as persuade them to loosen their grip. The little blighters can then be removed with tweezers as they appear. Second-line treatment - Occlusion/suffocation approaches. The fact that it was painless and there were lots of them and shaped like grains of rice are helpful pointers, but there are apparently more flies that can infest humans than I at first thought when you posted you query so I am not going to even hazard a guess. Also, I doubt after all these years if your memory will remember enough details to reliably identify the exact species but there is an identification aid linked to on this sight. Identification key to species of myiasis-causing fly larvae. After many years have past, the brain can size on details which were never really present at the time, so personal I would not bother to search further and from a clinical point of view it is unimportant, so even the treating physician may not know (or need to know) the specie in order to deal with it effectively.--Aspro (talk) 12:37, 30 November 2010 (UTC)[reply]

I'm reminded of the very strange condition suffered by Dede, the Indonesian "tree man". [5] [6]Steve Summit (talk) 12:44, 30 November 2010 (UTC)[reply]

Why shopuldn't you fill a hot water bottle from the domestic hot water supply?

I brought a hot water bottle that says in the instructions"Do not fill using water from a domestic hot water system as this can considerably shorten the life of the hot water bottle". Since it also warns against using boiling water the alternative is to part-heat a kettle. How is partly heating water in a kettle different from heating it in a domestic hot water system? The only difference I can think of is that the water from the hot water tap may have been kept warm for some time, but I can't see how this would make a difference. -- Q Chris (talk) 12:17, 28 November 2010 (UTC)[reply]

My loose guess is that it might have to do with impurities in the water — see, e.g. water softening. --Mr.98 (talk) 14:47, 28 November 2010 (UTC)[reply]
It's a wonder chemicalinterest has not answered this yet, is he asleep still :) It's the copper that's the problem with rubber. Case studies hot water bottle premature failure Obviously our article on hot water bottles requires this addition by a thermophilic editor who does not like to wake up in a wet bed.--Aspro (talk) 15:07, 28 November 2010 (UTC)[reply]
The original poster cited a British URL for the bottle. Many houses in Britain have water systems where only one (I hear) cold-water tap is fed directly from the mains, the hot-water system and other taps being fed from the tank. I can't find a Wikipedia article directly about this, but Water heating#Gravity system addresses it. Anyway, with such a system the hot water is in it longer and has more time to collect impurities. But I can't say if that actually is what they have in mind. --Anonymous, 03:33 UTC, November 29, 2010.
Yes, this is often the case in UK homes. The cold water comes straight from the mains, but if you turn on the hot tap, the water will come from a storage tank, commonly found in the roofspace/attic (although the water which comes out of the tap may be cold for some time depending on the vagaries of the heating system). This is why we were always told not to drink water from the hot tap, as it will be standing water, and tanks often have no lids. You can frequently tell the difference because the water pressure will be higher on the cold tap than the hot one. This page (a forum) contains some anecdotal information. See this from a national DIY chain which explains several kinds of residental water supply systems in UK homes. --Kateshortforbob talk 14:44, 29 November 2010 (UTC)[reply]

Bose-Einstein condensate of light

Many years ago I imagined an idea in a fantasy game, in which people would lay out spheres of "one-way mirror" in the sun until the light would "crystallize" into "flasheesh", a basic magical reagent better known for its recreational abuse by magic users who would shimmer and eventually spontaneously combust from their addiction. It seemed to me just about the most absurd thing I could come up with. Until that is today, when I read someone had done something about like this, and that the Bose-Einstein condensate of photons was being considered as a viable commercial process for solar cells and lasers![7]

The above story in Nature News is written to be very accessible, but even so it leaves me with a host of unanswered questions. Why can only a certain number of photons enter the cavity before thermal equilibrium is no longer possible? Where do the photons go, and are they "bound" to the dye molecules? Can you physically shove such a condensate of photons to change their wavelength? When they talk about a "super-photon", are they merely waxing poetic?

As you can tell, I haven't accessed the paper, but this looks like a big story, and I hope that the people who understand it will make a new article to explain it on Wikipedia. Is it time to call the DEA? ;) Wnt (talk) 13:56, 28 November 2010 (UTC)[reply]

The photons are trapped in a resonant optical cavity (basically imagine a box with mirrors on all sides, though this box is microscopic and two dimensional). Quantum mechanics selects for wavelengths that are integer fractions of the dimensions of the box. For example, if you had a 1 m box, then you could have 1 m waves, and 1/2 m waves, and 1/3 m waves, etc. The actual box selects for light in the visible spectrum. So that's one selection constraint. The other constraint is that the temperature of the gas should be in thermal equilibrium with the box. (The dye they used is basically there to promote the creation of thermal equilibrium quickly enough that photons can't escape from the box or be absorbed on the walls before equilibrium is achieved.) The energy per photon increases as the wavelength decreases. The result is that certain photon states are forbidden. For example, even a single photon in a high enough state would require more energy than the entire collection of photons at thermal equilibrium is supposed to have. Similarly, the resonant nature of the box forbids very low energy photons from being formed because they would require wavelengths much longer than the box. As you push more and more photons into the box they will try to reach equilibrium, but because certain energy states are forbidden you end up with an excess being forced into the lowest available state. It is this non-thermal excess in the ground state that characterizes the Bose-Einstein condensate.
As for what the result is like, the resulting ground state population is basically a laser. You get a coherent population of photons at a particular wavelength. The trick here is that the wavelength is determined by the physical properties of the box and not the atomic properties of the substances involved. This means you could tune the wavelength by adjusting the size of the box, which could have a variety of practical applications and allow for lasers at wavelengths which we don't have an effective means of generating now. It is also interesting that this system essentially transforms arbitrary light of sufficient intensity into a laser beam (with some unknown efficiency). That could have a variety of applications with respect to solar cells and other processes that require light concentration. Dragons flight (talk) 21:29, 28 November 2010 (UTC)[reply]
By "super-photon" they presumably mean many photons in the same quantum state. Lasers do that too, so "super-photons" have been around for a while. This looks like an interesting experiment, but popular science magazines have a habit of reporting everything as though it's going to change the world. Chances are this won't. -- BenRG (talk) 02:41, 29 November 2010 (UTC)[reply]
I think I follow what you mean by "lowest available state", but just to check: Photons are bosons not subject to the Pauli exclusion principle, and a Bose-Einstein condensate involves particles all in exactly the same state. So I assume that the lowest available state is simply the longest wavelength that can bounce back and forth between the walls of the cavity. This also means that despite my initial perplexed reception, the photons aren't really "stopped" somehow, but simply bouncing back and forth between two closely spaced mirrors.
But what I don't understand is how the behavior of one photon differs from many. I'd think that even a single photon with a wavelength not matching the walls of the box would somehow be forced to end up having some wavelength that fits, which might be the lowest available state. Again, as bosons, I don't see how the photons can affect one another. Unless the box somehow cools the photons - and if so where does the energy go?
This keeps raising all kinds of questions in my mind:
  • It makes me wonder what happens in an ordinary dye laser if the wavelength of the dye slightly disagrees with the spacing of the mirrors. Can you see the emission peak broken up into a set of integral wavelength multiples of the mirror spacing?
  • Can photons trapped in a box be switched between spin states? I think it could have spin 1, 0, or -1 hbar per photon, though I'm not sure if that adds up to much. Can you use a box like that as some kind of "photon gyroscope"?
  • Can you close off such a box and keep photons trapped inside for any length of time? How good can a mirror be when it is very small?
  • Can you use a device like this to capture ambient light and convert it into a single frequency best suited for driving a chemical reaction or other electrochemical process? (I suppose that's also what the solar cells are about) Do the antenna complexes of plants know this physics?
Wnt (talk) 17:00, 29 November 2010 (UTC)[reply]
A very clear explanation by dragons flight there. As for interacting photons, recall how a laser work. In some you have maybe one photon knocking an atom to a higher energy state, then a second photon comes along pushing it to an even higher state, then it decays sending out a third photon. So even though photons do not interact, they are causally connected. The two first photons "caused" the third one. I imagine that the photons "interact" through such middle-men in this setup.
To answear your bullet points: Photon gyroscopes already exist. Photons have spin +1 and this won't change that far as I can see. Nor can this collection of photons apparently be trapped any longer than other photons, but the light may "last longer" if there's a delay in re-emitting photons from the dye. As for converting ambient light into a singel frequency, that also exists today in many ways, such as fluorescence chemicals used as whiteners in detergents. So this might be one of many ways to achieve that.EverGreg (talk) 13:31, 30 November 2010 (UTC)[reply]

Water not freezing below the bridge

I was walking around in park (the temperature was 0 C or below) and noticed that the water below the bridge and in nearby area for some reason did not freeze (and the birds were swimming there). Why is that? —Preceding unsigned comment added by 89.77.158.172 (talk) 14:31, 28 November 2010 (UTC)[reply]

Currents. Kittybrewster 15:01, 28 November 2010 (UTC)[reply]
The bridge may provide shelter from the cold (e.g wind chill factor) and it may also be emitting heat absorbed over the summer / from usage which does just enough to prevent the water there freezing. ny156uk (talk) 15:11, 28 November 2010 (UTC)[reply]
The simpler explanation is that the open water has been able to radiate its heat into space but under and around the bridge it has been reflected back. This is why ice can form over-night under a clear desert sky, even though the upper air temperature is above freezing.--Aspro (talk) 15:43, 28 November 2010 (UTC)[reply]
Has it been snowing or frosting where you are in Poland recently? If so, it seems likely that they would have been out there laying down salt on the bridge, which is then running off or blowing off the bridge onto the ice below, melting it. Especially if the bridge is a road bridge and not a footbridge, and has a lot of traffic over it. WikiDao(talk) 19:32, 28 November 2010 (UTC)[reply]
As this is a common phenomena under structures which have not seen gritted, I take that explanation with a pinch of salt ;-) No one has mentioned yet, that the air temperature reported is only around the freezing point. Therefore, I was expecting someone to also say, that perhaps the water has not had time to stratify (remember science lessons at school about the density of water and what parts of a lake are the first to freeze and which parts are the last). The water may be a lot deeper around the bridge which would be the reason why that spot will be the last to ice. Here is a link Understanding Pond Stratification. We need the OP to come back with some more observations.--Aspro (talk) 21:04, 28 November 2010 (UTC)[reply]
Beneath what other structures does this commonly occur? WikiDao(talk) 21:22, 28 November 2010 (UTC)[reply]
Roofed over boat moorings on pleasure lakes is one example. Common in the Victorian period before people could stay in all night and watch I love Lucy. --Aspro (talk) 21:40, 28 November 2010 (UTC)[reply]
An alternate explanation is that they have installed some sort of heaters around the bridge to stop the water from freezing. Having ice directly freezing against the bridge supports could have a detrimental effect on the life of the bridge. They may be deliberately stopping the water from freezing... --Jayron32 21:02, 28 November 2010 (UTC)[reply]
Is a common feature of these cases that water is moving near some fixed object (pier, etc.) and/or something is moving in the water (birds paddling around, etc.)? I've often heard folklore (yeah yeah [citation needed]/[original research?]) that turbulence makes water freeze more slowly, so water lapping at a piling would freeze after a still pond or a smooth gently-flowing stream. Sea ice#Formation of sea ice suggests this isn't a complete crock either. DMacks (talk) 22:07, 28 November 2010 (UTC)[reply]
Folk can be right too. Turbulence warms water and Mechanical equivalent of heat is your citation. Cuddlyable3 (talk) 22:47, 28 November 2010 (UTC)[reply]

Growing bacteria, beef bullion

I'm trying to grow bacteria with unflavored gelatin in petri dishes. I've heard to add beef bullion to the boiling water, then let it dissolve. Beef bullion is high in sodium, and sodium was used as a preservative in meats. Would sodium inhibit the growth of bacteria? Also, how would you identify the bacteria that grows, and what surfaces would bear the most bacteria? Albacore (talk) 17:00, 28 November 2010 (UTC)[reply]

Different bacteria require different Growth mediums. Once you know what you what to grow, you can choose a suitable source to provide the nutrients. Some of these bacteria can be very dangerous in such large quantities that end up in culture dishes -so beware. --Aspro (talk) 17:09, 28 November 2010 (UTC)[reply]
I'm really concerned, now that I've thought about it, that you are going to try doing this in the house – don't! Here are two links you might find useful: Safety Guidelines & MICRO-ORGANISMS FOR EDUCATION. The salts (sodium) in the beef bullion doesn't matter. After all, its best diluted down to half the strength normally used for making stock. At that level the salts will have little inhibitive effect. --Aspro (talk) 18:37, 28 November 2010 (UTC)[reply]

Geologic Time Scale

Found in the discussion of geologic time is a Geologic Time Scale. Where can a full size copy of this document be acquired?

Rolf Olson Salem, OR —Preceding unsigned comment added by 24.22.3.126 (talk) 18:44, 28 November 2010 (UTC)[reply]

Other than Geologic time scale, what exactly are you looking for? BTW: End the Holocene now, ask me how. Hcobb (talk) 18:54, 28 November 2010 (UTC)[reply]
How? WikiDao(talk) 19:38, 28 November 2010 (UTC)[reply]
Oh! Don't you read all of Wikipedia from A to Z? You don't need to do nothing, as it finishes in two years time.--Aspro (talk) 21:15, 28 November 2010 (UTC)[reply]


There are several diagrams on the discussion page. Do you mean this one:[8]. If so you can use the right-hand download tool. --Aspro (talk) 19:02, 28 November 2010 (UTC)[reply]
Your options for obtaining a large size print of a digital image are a) Tiled printing or "rasterbating" using an ordinary A4 PC printer, or b) use a printing company that is equipped to produce large placards. If you just need to present a large image, e.g. for a classroom lecture, consider using a Video projector instead of a paper document. Cuddlyable3 (talk) 09:03, 29 November 2010 (UTC)[reply]
For most of the pics, you first click on it, then click on "Full resolution" underneath to get the full image. If you mean the small Graphical timelines wiki table, a screen shot should work there. If you mean the much larger Table of geologic time, then the best I can do is to break it into 3 screen shots and prints those, since it's not a pic but rather a large wiki table. I have a max screen resolution of 1920×1200, but someone with a higher res might be able to print it all on one sheet, although the writing would then be tiny. Would you like us to provide you with it as 3 pics, so that you can print it, or do you know how to do that ? StuRat (talk) 15:02, 29 November 2010 (UTC)[reply]

Skinny vs baggy jeans in freezing weather

Being a student up here in Oswego, I know a lot about walking around all day in freezing wind/death snow. But this question has been seemingly unanswerable by my friends and I. What would be, according to science (if it is science that applies here), warmer, skinny jeans or loose, normal fitting jeans? Skinny jeans are closer to the skin, and seem to not let any of the cold air in, but because they are so close to the skin, the cold air gets closer to the skin, too. Looser jeans seem to have more room for the cold air to circulate around your skin. I've clearly tried to think about this for a long time! The jeans are both made of the same fabric, so that shouldn't be an issue. ?EVAUNIT神になった人間 19:00, 28 November 2010 (UTC)[reply]

Yes, you need some trapped air. A pair of these underneath would also make all the difference. After all, if they're good enough for Clint Eastwood, they should be good enough for you. --Aspro (talk) 19:09, 28 November 2010 (UTC)[reply]
Looser will be better on the basis of allowing sweat to escape, and on the same note will allow hairs to stand out more. Also, what heat is emitted by your legs will travel more freely up the wider pants, creating a chimney-effect (if rather negligible). The baggier jeans will also be made of more fabric. In addition to this, any consideration of moisture favours baggier jeans: Skintights that grow wet will immediately cool down parts of your legs, while baggier jeans can keep this moisture at a range. 88.90.16.188 (talk) 19:11, 28 November 2010 (UTC)[reply]
Not answering the question, but there exist flannel lined jeans, such as these jeans from L.L.Bean (not spamming for them, but that's where I got my pair). They are quite comfortable in colder weather, though if they do get wet, they take forever to dry. Back on topic, tighter jeans may restrict circulation in extreme cases, which would be undesirable in colder whether. Buddy431 (talk) 19:34, 28 November 2010 (UTC)[reply]
Definitely looser. The amount or air that comes in because the opening is larger is nothing compared to the insulation the air gap gives you. Ariel. (talk) 20:55, 28 November 2010 (UTC)[reply]
Best to wear baggy jeans with layers underneath, such as jogging trousers, long underwear, leggings, or all three. Insulation is provided by trapped air, so loose layers are best. 92.15.14.132 (talk) 22:49, 28 November 2010 (UTC)[reply]
Old-style jodhpur pants may be best. They were baggy at the top (to allow air circulation inside) but narrow at the ankle (to prevent cold air from getting in). They were actually designed for horse riding, not for keeping you warm, but this would be a nice side effect, if made of the proper fabric (which would wick moisture away, such as cotton). We don't have a good pic of the old-style version in our article, so here's one from somewhere else: [9]. StuRat (talk) 14:45, 29 November 2010 (UTC)[reply]
Jeans are utterly unsuitable for cold weather. Get something in a nice woollen-worsted instead. DuncanHill (talk) 14:53, 29 November 2010 (UTC)[reply]
Looser jeans with a relatively tight bottom will do the best, I think the style is called "boot cut". Tight jeans just get the cold air closer to your leg. From my OR though, don't worry about your jeans, concentrate on covering your head (toque), neck (scarf), hands (gloves) and feet (socks'n'boots) as that is where the most heat loss (head & neck) and discomfort (hands & feet) occurs. Spend the big bucks on those accessories and they will last for decades. Franamax (talk) 21:56, 29 November 2010 (UTC)[reply]
One thing to bear in mind is that denim is a poor insulator, fabric-wise, as it is quite loosely woven and has a high wind chill factor. ([10], [11]). --Kateshortforbob talk 09:55, 30 November 2010 (UTC)[reply]

Sleep, tiredness, exercise

1) Why does taking a lot of exercise in the day make you sleep well that night? 2) Why does not getting enough sleep make you feel tired? You havnt been exercising all night, so why should you feel tired? 3) Why is the tiredness you get from not sleeping enough the same as the tiredness you get from exercising a lot? Do they have any differences? Thanks 92.15.14.132 (talk) 23:10, 28 November 2010 (UTC)[reply]

Have you looked at the articles about Fatigue (medical), Physical exercise and Sleep? Cuddlyable3 (talk) 08:52, 29 November 2010 (UTC)[reply]

I have now thanks, but they don't answer the questions. 92.24.176.72 (talk) 13:39, 29 November 2010 (UTC)[reply]

1) Exercise causes "damage" to the body, which must be repaired by additional sleep. I put damage in quotes, because it's not the usual permanent damage you might think of, but rather easily repaired damage. For example, micro-cracks occur in the bones, which are then filled in with more bone, strengthening them even more than before. Muscle cells are also repaired and expanded.

2) Even without exercise, some damage occurs (especially in the brain and eyes) and sleep is needed to repair that. So, if you don't get enough, your body tells you more is needed, by having you feel tired.

3) I'd expect the tiredness from exercise to be more in the muscles, tendons, and bones, such as aches, and regular tiredness to be more in the brain and eyes, in the forms of mental slowness and eye strain. StuRat (talk) 14:31, 29 November 2010 (UTC)[reply]

Circular logic

I had to do an experiment designed to show the "equivalence of mechanical work/energy and heat". This was done by heating a piece of copper with friction. The amount of work was calculated, and change in temperature of the copper was predicted using the heat capacity of copper. This was compared to the actual change in temperature of the copper.

But if the heat capacity of copper is determined experimentally by how much energy it takes to raise a mixed mass of copper one degree, then isn't the whole experiment circular? 70.52.44.192 (talk) 23:16, 28 November 2010 (UTC)[reply]

But the heat capacity of copper could be determined using chemical energy, like gas. Vespine (talk) 00:23, 29 November 2010 (UTC)[reply]
Or by Resistive heating, where the amount of energy put in can be measured quite accurately by maintaining a certain (known) current over a certain (known) voltage drop for a certain (known) amount of time. Buddy431 (talk) 01:50, 29 November 2010 (UTC)[reply]
...and thus you don't need to know the heat capacity of the piece of copper, merely the electric energy V*I*T that causes the same temperature rise as the frictional work. Cuddlyable3 (talk) 08:46, 29 November 2010 (UTC)[reply]

November 29

How well does heterodyne frequency downshifting preserve phase?

When an optical heterodyne is used to shift a terahertz signal to a lower frequency, how accurately is the original phase information preserved in a form which would be useful for interferometry? I'm particularly interested in 9-10 micron infrared; presumably that would use a CO2 laser heterodyne -- but to which frequencies makes that range most useful for accurate interferometry, and how accurate does it turn out to be, in quantitative terms? I've read the Heterodyne and Laser Doppler vibrometer articles, [12], section III on p. 1253-4 of [13] and [14] and can't determine how well this works. Ginger Conspiracy (talk) 04:19, 29 November 2010 (UTC)[reply]

The phase will be critically dependent on the phase stability of the heterodyne. This can be measured nu the phase noise, measuring the spectrum of the supposedly stable oscillator. The idea will be to get a frequency offset you are interested in and make sure that the level of spurious signal is sufficiently below that of the carrier. However if you use the same oscillator signal to mix down with two or more points of a coherent signal, and then take the difference you will cancel out most of the noise introduced by an unstable oscillator. A 10 micron infrared is well beyond what I would term a terahertz signal. The quality that you would need is a narrowband enough heterodyne that when its spectrum is superimposed (convolved) on the desired downconverted signal you get no undesirable effects. Graeme Bartlett (talk) 08:08, 29 November 2010 (UTC)[reply]
Thanks very much. Phase noise is exactly what I was looking for. It's always a great pleasure to see such seemingly esoteric articles with such great content and external links. Ginger Conspiracy (talk) 03:35, 30 November 2010 (UTC)[reply]

Modified corn starch

When the ingredients listed for a food product include "modified corn starch", does that mean starch that was taken from corn and then modified, or starch that came from modified corn? NeonMerlin 06:00, 29 November 2010 (UTC)[reply]

The former. The starch is modified. The corn may (or may not) be modified, but that is irrespective of the name. See Modified starch. --Jayron32 06:04, 29 November 2010 (UTC)[reply]

LED Xmas light string efficiency

Thinking of buying some, and have a few Qs:

1) How does their efficiency compare with the following:

a) Incandescent Xmas light strings.

b) Regular incandescent lights.

c) Regular LED lights.

d) Regular CFL lights.

Efficiency being defined, of course, as lumens of light produced per watt of power used, but feel free to use other units, if you prefer. What I'm getting at with efficiency is knowing whether my electric bill will go up or down if I largely light the house with those strings instead of with regular lamps and overhead lights.

2) Also, how long should a string of LED lights last ? And when one light fails, will the rest continue ? Is there some point at which too many lights have failed, where the whole string will go out ?

3) As for safety, I assume that they use less energy and produce less heat than incandescent Xmas lights, so this should reduce the risk of fire, right ? StuRat (talk) 14:20, 29 November 2010 (UTC)[reply]

You are comparing mono-chromatic light to white light. The theoretical max for white light is 251 lm/W, but for a single color it's 683 lm/W (it varies a bit depending on the color). This is not just theoretical - it makes a huge difference in practice too. Since you need colored lights, LED is by far, no question about it, the most efficient. Incandescent, besides being inefficient, and hot, also looses a ton more efficiency by using a filter to make it colored. CFL can be made in a single color, but in practice it's not.
Re-reading what you wrote, are you trying to use them for general lighting? If so then CFL is best. See Luminous efficacy for chart. LED has a lot of vocal support, but it's not as efficient as CFL, and the color quality is worse too.
LEDs last about 20,000 hours, but they don't burn out, they just get dimmer. LEDs are probably wired in series. An LED needs about 5 volts, so you have about 24 of them in series for 120 volt service. I'm not sure of the failure mode of an LED (i.e. does it fail open or closed). But normally they don't fail anyway, unlike incandescents.
I'm not sure there is a lot of risk of fire from incandescents anyway (I mean you could, but in normal use it's not a problem) so that's not really a comparison. But less heat does mean a lower A/C bill. Ariel. (talk) 21:01, 29 November 2010 (UTC)[reply]
I'm not so sure about some of your claims.
For example the colour quality claim. For generic 5mm or similar LEDs sure. But high CRI (90+) LEDs do exist (see [15] for example). These tend to exist in warm white too although that's (IMHO) strong evidence that's a cultural thing so not really relevant. I don't know how these compare to state of the art CFLs but most people aren't using those (i.e. they seem satisfied with the lower quality) so it seems fair to point out you can get something which will probably be good enough if you really want. Of course CRI probably isn't perfect anyway [16] although that's somewhat of a field beyond me.
I don't know about the claim of LEDs not failing either. The 20k lifetimes are great in theory. But from what I've read many LEDs lights aren't living up to their purported lifes. In particular one problem is given the common high variance in Vf if these are matched into parallel arrays without proper regulation, some LEDs tend to be overdriven, these eventually fail and then the others are then overdriven so the how thing quickly fails. [17] [18] [19] I've also read of very quick and rather dramatic dimming of cheap Chinese white LEDs (mostly the 5mm etc kind) even with rather low non over-driven currents. LEDs have a big problem in they really don't like heat, it reduces the efficiency, changes the colour and drastically shortens the lifespan, so incandescent bulb replacements are still problematic. (Enclosed fixtures are usually a case of just don't get, except perhaps when it's purposely designed for LEDs.)
In terms of luminous efficacy, while CFLs probably still win here I don't think they will stay much longer, our article currently has 93 [20] which is over a year old. This compares to a best of 120 for CFLs from our article. But LEDs are advancing at a far faster rate AFAIK (compare the Cree XM-L LED launched recently with the XP-G launched last year for example [21], while these are raw LEDs the bulbs are clearly going to trend). Of course if you want high CRI or warmer whites you're worse off and I'm not sure how LEDs luminious efficacy compare with CFLs when at that level.
Price wise LEDs still have a way to go, particular since as I mentioned they may not last as long as sometimes claimed.
Nil Einne (talk) 22:05, 29 November 2010 (UTC)[reply]
What you wrote doesn't really dispute my claims. I have never understood why people keep saying LEDs have better color, CFLs and LEDs make colors by the exact same method! LED's have no advantage here. They make UV and phosphors change the color. CFLs do exactly the same thing. The difference is that CFLs generate a deeper UV, which helps a lot. I simply check the packages and displays in stores, and it's no contest - CFLs are far better. That post you linked says essentially "LEDs are not doing so well in color accuracy, so lets redefine color accuracy so LEDs do better".
On top of that color accuracy and efficiency are enemies of each other, so it doesn't help to compare a high efficiency 70cri led. Notice the Toshiba bulb: 93 lm/W is at 70 cri. If you want 90 cri it's only 69 lw/W which is much worse than a CFL for that CRI. The 160 lm/W is only at very very low power levels, and no mention is made of CRI. I don't agree that LEDs are advancing faster. They are advancing fast in total lumens, but in efficiency it's at a much slower pace (and remember low cri doesn't count). Maybe one day LEDs will be better, and I'm glad there are people funding the research by buying poorly performing LEDs, but I'm not going to (OK, actually I did buy an LED for a bedside lamp for kids, since it's unbreakable :). Ariel. (talk) 23:29, 29 November 2010 (UTC)[reply]
Long semi OT reply to IMHO questionable claims Nil Einne (talk) 18:32, 30 November 2010 (UTC)[reply]
Who said they have better colour? I didn't. Nor did StuRat. You however did claim they have worse colour, which I stick by my claim is questionable. For starters there are high CRI LEDs. Are these worse then the best CFLs? I don't know. You definitely haven't presented any real evidence that there are. Your anecdotal evidence is somewhat irrelevant since 1) People have different preferences 2) You haven't indicated how you compared, did you use colour swatches? Was any semblence of a blind test done 3) We have no idea what sort of LEDs were being compared, were high CRI LEDs even involved? Nor is it clear what CFLs are being compared. Now it may be that there aren't any real high CRI replacement bulbs, LED bulbs are still somewhat of a niche product as I've indicated. And as I've said, AFAIK the evidence suggests most people don't care that much about CRI so it's hardly surprising that of what does exist, manufacturers are concentrating on what's good enough for the majority rather then what a minority at the current time may want.
That bit about people not caring that much is of course an important point. I admit I don't have clear evidence here but as I already said, most CFLs aren't that high CRI. In NZ the CRI isn't usually even advertised or specified from what I've seen. High CRI CFLs do exist but they tend to be niche products AFAIK. Now as I said, I don't have clear evidence like sales figures so I could easily be wrong, but I don't think I am. Given this, the claim that LEDs have worse colour is even more misleading since from most peoples POV you can get better CRI LEDs then what they're currently using in CFLs. This doesn't mean LEDs have better colour, I never said they do, it just means it's misleading to say CFLs have better coloru as you have done repeatedly.
As for the CRI definition thing, well I said it's somewhat beyond my expertise. However I freely admit I trust actual published research, much more then I do claims by random wikipedians backuped by nothing so I'll trust the research suggesting CRI, something developed in the 30s and 60s may not be perfect. Even more so when the only apparent response said wikipedian has is that "LEDs are not doing so well in color accuracy, so lets redefine color accuracy so LEDs do better" yet some of this research was from 1995 and attempts were made to reform CRI itself in 1991-1999. Blue LEDs had started to gain interest by 1995 so I guess it's possible that people in 1995 were trying to fix the standard to give LEDs a leg up again CFLs but I admit wacky conspiracy theories have never been my thing. (This is not to say I'm denying such things tend to be very political our article itself says "The committee was dissolved in 1999, releasing (CIE 1999), but no firm recommendations, partly due to disagreements between researchers and manufacturers").
Ironically from what I've read one complaint possibly about CRI or at least luminous efficacy is that it underemphasises the blues which makes me wonder about your UV claim. I'm also a bit confused about the relevance of your anecdotal evidence anyway, surely all that matters if you think CRI is perfect is the CRI and colour temperature (and luminous efficacy), why worry about how they look? If you get equivalent CRI and colour temperature to what you prefer CFL and LEDs then these are the same and just compare the efficacy. Unless you are acknowledging that CRI isn't perfect.
Note I already said you lose efficiency if you want increased CRI (or lower colour temperature) so I'm not really sure why you're repeating what I said. You've claimed a lower CRI LED is much worse then a low CRI CFL but provided no evidence. But rather then waiting for some, let me try to find something. For starters your claim is misleading anyway since the 69 lumens per watt is warm white not cool white which the 70 CRI is so it isn't just the higher CRI but the higher colour temperature. The 69 lumens per wat BTW is also 80 CRI not 90 CRI. But looking for highish CRI CFL, I was directed to (well the main site) [22] where we see EnergyMiser warm white 600 lumens 80 CRI with 55 lumens/w (ooops!). But moving on perhaps that's not totally fair since there's also a 68 lumens per watt 820 820 lumens warm white 80 CRI and a Phillips 700 lumens warm white 82 CRI 64 lumens per watt and uh lets ignore the TCP. The Toshiba doesn't specify what they mean by warm white so I'm presuming of course they mean 2700K like the others but I'm not that sure (I did find some claims the Toshiba was 2700K but none from Japan). While the CFLs here don't actually come out better I'm not suggesting these have the highest efficacy so I'll concede as I did in my first post CFLs probably still win. Particularly since, I've seen (a while before this this post) some evidence the Toshiba figures may be somewhat inflated (although I've read that's not uncommon with CFLs either) and I've also read that the Toshiba's in Japan have shit power factor and other issues because of lax standards in Japan and meeting the stricter standards in the EU and US lowers efficiency (I'm presuming the CFLs meet the stricter standards since that's a US site) [23]. (Another problem is of course heat as I've already emphasised, your LED bulb may be 600 lumens when you first turn it on, I wonder if it will still be that 8 hours later in a low airflow room and a somewhat enclosed space.) However as you may have guessed, I am awaiting support for your claim 'which is much worse than a CFL for that CRI'.
You're right that low currents LEDs have greatest efficiency at low currents, this was perhaps not something I made clear enough in my original post. I'm ignoring your irrelevant claim that low CRI doesn't count since as I've already said while your welcome to your POV, there's no evidence it's held by the majority of people. The XM-L is still not really widely available so it's difficult to get good comparisons but from what I've seen 10% efficiency gain isn't unresonable. 10% may not seem like much to you but I'm sure quite a lot of industries will be happy with a 10% efficiency gain a year. I'm not aware CFLs are advancing at anything close to this rate.
P.S. One of my earlier comparisons was a fluourescent light not a CFL which perhaps wasn't the fairest but since these are commercial available bulbs (as opposed to raw LEDs) I'm still willing to let it stand. I would note our luminous efficacy article suggests 46–75 for CFLs.
P.P.S. The website I used above showed CFLs all had an advertised CRI of 80 or higher (although I was directed there when looking for highish CRI). My impression had been that many CFLs have a CRI of about 70-75. However I appear to be mistaken on this, for example I found [24] which shows all CRIs tested there (Australian) were 80+ and it is in fact the ANZ standard. I also find [25] which suggests ~80 is the norm. In that case, and given a CRI of ~70 seems the norm for cheap or generic LEDs I'll concede the average LED is worse then the average CFL. However it still doesn't change the fact you can get LED light bulbs better then the average CFL and in fact approaching the best CFLs e.g. [26]. (Efficacy wise that's 54 lumens/watt so not that great although I still don't know what the CRI for high 90+ CFLs are. Note that the Australian study I linked to found the luminous efficacy was around 60-80 lumens/watt for the 80.4-87.7 CRIs in that study so I'm still not seeing much evidence for these far better efficacy of CFLs that was alleged. These are real world independent tests however from my brief read thorough I think they found most manufacturers live up to their claims so I'm not sure if the LEDs are going to be much worse. Perhaps this is a matter of semantics, I don't consider 20-30% far better, particular considering the rate of advance of LEDs but perhaps you do. )
Nil Einne (talk) 18:32, 30 November 2010 (UTC)[reply]
Maybe high CRI, high efficiency LEDs exist, but they are certainly not commonly available - I always check the lamp displays, and all the LEDs on display are very blue. And many of the LEDs don't even list lumens, which I find quite shady. Additionally the CFLs will say "warm/cool white", etc. The LEDs don't, they leave you to guess. I think you are arguing about LEDs that exist, and I am arguing only about those that I can find in a hardware store. Right now, if you go to buy a bulb in a store, CFLs beat LEDs, but I'm willing to accept that it's possible to buy great LEDs online. Ariel. (talk) 19:28, 30 November 2010 (UTC)[reply]
Ok thanks for the clarification. I suspect however it depends where you live, I belief for StuRat this is the US so perhaps your comments are somewhat appropriate for him. But it wouldn't surprise me if the Toshiba linked above was available in some Japanese hardware stores. In NZ I don't think I've even seen a mass market retail store selling LED bulbs, nor in Malaysia (although haven't looked that well there, it's not something that interests me) so if you want LED bulbs buying from specialised stores is probably your only choice. Some of the bulbs, e.g. the Cree LR6 aren't really intended for the mass home market anyway given heat, cost and perhaps weight issues.
Note in case this wasn't clear enough I wasn't trying to suggest LEDs are better at the moment, just that in terms of CRI, CCT and efficacy I don't think they are really that far from CFLs. Realisticly, the number of people interested in buying $100+ light is small (particularly given the care needed, the small number of advantages and the fact 5-10 years from now people will be laughing at the old prices) hence as I mentioned they still have a long way to go on price.
Nil Einne (talk) 20:12, 30 November 2010 (UTC)[reply]
(EC) Semi answering my own question I find [27] with a 13W CFL with CRI 93+, 780 lumens, 5500K CCT. That's 60 lumens/W in other words the Cree LR6 linked above doesn't seem that bad in comparison particularly noting it was launched in 2007 or 2008 from what I read (not sure whether it's undergone improvements since then). Nil Einne (talk) 20:12, 30 November 2010 (UTC)[reply]

Thanks for the answers so far. I had strings of LED Xmas lights in mind for two purposes:

A) Night lights. Should be more efficient than the incandescent night lights I use now. These are just so I can find the light switch without smashing my toe on the coffee table, etc. I do have one CFL night light, and I will probably keep that one.

B) Area lighting. I don't like having a single bright spot in the room, which either an overhead light or table lamp produces. Strings of Xmas lights would give a more even lighting.

As for the color, I'm flexible there. If I wanted white, overall, couldn't I combine different colored LED lights to get that ? StuRat (talk) 05:24, 30 November 2010 (UTC)[reply]

For evenly lighting a room, you might consider up-lighting. We have a room that was previously lit by small lights all 'round the walls, and now we have a bright free-standing lamp that directs its light in a cone facing the ceiling. It's maybe 4 or 5 feet lower than the ceiling. The light reflects off the white ceiling and evenly lights the whole room very satisfactorily. It's very impressive if you haven't tried it before. 86.161.109.130 (talk) 11:11, 30 November 2010 (UTC)[reply]
I can't offer that much advice since it's not something I've explored that much (I'm more interested in the flashlight arena). You may find some advice here [28] although be warned I've sometimes found the culture there fairly elitist and pro-American/anti-Chinese. However I've also seen a lot of fairly complex discussions about lighthing including custom solutions. (I'm not sure whether that's really what you're going for since it's likely to be rather expensive.)
My guess is using the white locations and right lens, reflectors etc will produce a far better result then a lot of small Christmas lights.
You can get RGB LED lights like this [29] but they tend to be orientated to having different colours rather then mixing the right white light for your purposes (although you may be able to do that, I don't really know). They also tend to be expensive, low powered and I expect not that efficient. I'm not particularly sure they give great CRIs either whatever you do. I suspect just having different colour lights won't work very well unless you are going for a disco/nightclub effect.
Nil Einne (talk) 18:50, 30 November 2010 (UTC)[reply]
Hmmm, "the white location" ? :-) StuRat (talk) 22:40, 30 November 2010 (UTC)[reply]

Computer modelling of Artificial consciousness

"Computer modelling shows that even consciousness can be generated with very small neural circuits....only a few thousand could be enough to generate consciousness." from http://www.sciencedaily.com/releases/2009/11/091117124009.htm

What computer modelling is this? I've tried searching. Thanks 92.24.176.72 (talk) 14:32, 29 November 2010 (UTC)[reply]

It's probably an artificial neural network. I've taken the liberty of cross-linking this to The computing ref desk. CS Miller (talk) 16:15, 29 November 2010 (UTC)[reply]
That popular article was based on this scientific paper. Discussion of artificial neural networks in that paper starts on page 7. The popular article is apparently being sloppy and exaggerative when it refers to the kind of cognitive tasks performed by artificial neural networks as discussed in the paper as being "consciousness". The paper itself does not use the word "consciousness". Red Act (talk) 16:25, 29 November 2010 (UTC)[reply]
Which is funny because "consciousness" is precisely the wrong way to describe what is going on. It's not surprisingly that you can make a small neural network count; counting is computationally easy. When humans count, they don't have a special counting neural network that they can invoke. If you tell someone to count by threes, or to stop counting and start again later, writing their number down so they don't forget it, they'll be able to seamlessly make the appropriate changes to their mental algorithm. So humans are able to reflectively modify their own cognition. Consciousness, normally defined as awareness of self, is the hard part. Executing little algorithms is easy, regardless of the substrate (neural networks, the symbol-manipulation of intelligent brains, or assembly language). Writing them is hard. A bee brain may be capable of counting, but I really doubt they can decide to count, or figure out how to count. Paul (Stansifer) 16:55, 29 November 2010 (UTC)[reply]
The problem of course is that we don't have a good operational definition to test for consciousness. This problem is discussed in our testing section of the Consciousness article. Nimur (talk) 19:00, 29 November 2010 (UTC)[reply]

Role of salt in ethanol precipitation of DNA

I don't understand the role of the salt. Does it cause the phosphate groups in the DNA to dissociate, giving up a proton? Or else if the the phosphate groups are already in the deprotonated form, why is the salt needed? Gidip (talk) 18:18, 29 November 2010 (UTC)[reply]

I was told it was because you needed a counter ion for the phosphate groups. In solution at neutral pH, the protons are already dissociated from the phosphate groups. Because of electrostatic repulsion, you can't force just the charged DNA into the solid phase - you need to counter the charge with a positive ion. The salt solution provides enough counter ions for the DNA to come out of solution as a neutral salt. (Note that DNA behaves the same as any other ionic species in this regards.) -- 140.142.20.229 (talk) 18:36, 29 November 2010 (UTC)[reply]
Well heck, we've got an ethanol precipitation article. DMacks (talk) 20:33, 29 November 2010 (UTC)[reply]
The net charge in the system has to be zero. Therefore, for every negatively charged dissociated phosphate group there is a proton. Then what's the salt for? Gidip (talk) 08:10, 30 November 2010 (UTC)[reply]
If they have a high propensity to dissociate (i.e., acidic), you would have to lower the pH to make that happen effectively. But you're right, if the solvent becomes non-dipolar enough, they would be more likely to stick. But they would stick as a covalent OH (not just ion-paired). Neutral covalent compounds are generally soluble in organic solvents, so you haven't accomplished the precipitation: the goal is to make the DNA insoluble, not just into some net neutral complex. So instead, seems like the goal is to promote formation of ionic salts of the phosphate. DMacks (talk) 09:16, 30 November 2010 (UTC)[reply]

Red bleach stains

Using laundry bleach, I occasionally splash some on my clothes, and get a stain. Typically the stain is lighter than the fabric, but I've recently encountered some odd exceptions:

1) A black pair of jeans had bright red dots where it was splashed. I guessed that the black color was, in fact, made up of multiple colored dyes, and that the red component was more bleach resistant than the others.

2) A white knit shirt, with a collar made of a different material, had the white collar turn pink (a pale rust color, to be more specific), while the rest of the shirt was unaffected. I can't use the previous explanation here. The rust color makes me think that there was actually some iron in the collar, which then oxidized when exposed to bleach. Is there any white dye which contains iron ? StuRat (talk) 20:55, 29 November 2010 (UTC)[reply]

I think your explanation for 1) is correct. As for 2), was the collar uniformly pink, or in a characteristic `splash' shape? If the former, I propose color was released from other fabrics, and preferentially bound with the collar. My understanding is that white cloth is not generally made these days by dying with white dye, but by bleaching off-white source material. SemanticMantis (talk) 23:04, 29 November 2010 (UTC)[reply]
2) It's the splash shape, any ideas ? The shirt might be very old, I inherited it. Also, the red showed up immediately when the bleach hit it. StuRat (talk) 05:07, 30 November 2010 (UTC)[reply]

Fusion and fusion

what I don't understand is were this energy comes from. Here's what I mean. The reaction of deuterium and tritium goes like this:
2H + 3H → 4He + 1n + energy
Were does this energy come from? You are left with less mass than you did before, but you still have three neutrons and two protons, just like before, yet they are lighter. I don't see how you can get "light particles"; a proton is always a proton, with three quarks, right? --T H F S W (T · C · E) 21:42, 29 November 2010 (UTC)[reply]

I believe the answer is in the Binding energy. Vespine (talk) 21:57, 29 November 2010 (UTC)[reply]
Then why is there mass loss? Does the binding energy hover around the particles like dark matter around a galaxy? --T H F S W (T · C · E) 22:11, 29 November 2010 (UTC)[reply]
The loss of mass and release of energy are the same. I think this is ultimately the same reason a compressed spring has more mass than a spring at rest, i.e. E=MC^2. SemanticMantis (talk) 22:57, 29 November 2010 (UTC)[reply]
Exactly. If you need a particle to imagine being converted into energy, the mesons mediate the binding energy of the nuclear force, but they are very short lived, virtual particles, so it may make more sense to just think of conservation of energy. Ginger Conspiracy (talk) 23:14, 29 November 2010 (UTC)[reply]
I suspect the difficulty in visualizing this is because the popular image of "energy" is as a little lightning bolt, i.e., electrical energy. "Energy" is a far more diffuse and diverse concept though. Personally I think our way of visualizing nuclear reactions is harmed by our persistent representation of the energy released as a little lightning bolt, when it is really released as kinetic energy in the speed of the constituent parts. In any event, thinking of energy as a "thing" that hovers around (or zaps out) is going to mislead you every time. --Mr.98 (talk) 00:19, 30 November 2010 (UTC)[reply]
To answer the question, the energy in this case is almost certainly released as heat energy; the fact that the alpha particles are moving after being released means they have energy. The speed at which they move will be ultimately related to the amount of energy released by the reaction. Heat energy is just the energy of molecular motion. --Jayron32 04:14, 30 November 2010 (UTC)[reply]
The nuclear binding energy is actually negative, but that's not important; what's important is that the binding energy per nucleon is different for different elements (and isotopes). If you plot it as a function of nucleon count, you get a bowl-shaped curve with the lowest point at Iron-56, as shown here (upside down). Thus, you can liberate energy by putting small atoms together or by splitting big ones apart. -- BenRG (talk) 08:34, 30 November 2010 (UTC)[reply]

is this true?

http://i.imgur.com/GBGAY.gif
can you really build it like that or is it just an animation? 82.234.207.120 (talk) 21:43, 29 November 2010 (UTC)[reply]

Kentucky Do-Nothing. Plans are here. Here's the Facebook page. Buddy431 (talk) 21:49, 29 November 2010 (UTC)[reply]
Thanks! Now I finally have a name for those things. I had one when I was a kid but never knew the name. Dismas|(talk) 07:12, 30 November 2010 (UTC)[reply]
Also known as a "Do-Nothing Machine" or "Bullshit Grinder", or more formally, as a "Trammel of Archimedes". When used to draw ellipses, they may be known as an ellipsograph. Buddy431 (talk) 20:11, 30 November 2010 (UTC)[reply]
I see that when the angle of the rod changes uniformly, there is simple harmonic motion in both axes. Is there a simple way to regulate the motion of the device so that it traces out an elliptic orbit around one focus according to Kepler's first law? (Despite some material at orbit equation, eccentric anomaly, true anomaly, etc., I should say that deriving this from scratch doesn't look easy...) Wnt (talk) 11:16, 30 November 2010 (UTC)[reply]
What exactly do you want it to do? You can easily change the size and shape of the ellipse it produces. See this demo. Buddy431 (talk) 20:14, 30 November 2010 (UTC)[reply]

Too many birds on a power line

Yesterday, I looked outside and saw that the power line to our house was completely filled with ravens. This wire is about 100' long, and was sagging quite a bit from the weight of the birds. I've lived here 17 years, and have never seen that many birds on one power line before. I have two questions: 1) what would cause that many ravens to congregate on my power line, and 2) is it possible for the birds to actually break the wire? Shuttlebug (talk) 21:44, 29 November 2010 (UTC)[reply]

The wires are strong enough for a person to hang from them, so I doubt birds could hurt it. Ravens like to group. I've frequently noticed large flocks right before storms. Ariel. (talk) 22:06, 29 November 2010 (UTC)[reply]
Probably planning a murder. HalfShadow 22:07, 29 November 2010 (UTC)[reply]
groan. DMacks (talk) 22:10, 29 November 2010 (UTC)[reply]

(edit conflict)Question1: I suspect that it was either 1) a group of young birds or 2) a group of males looking for females.

Question2: No. Metals, such as those cables are made of can be stretched and bent a long way before snapping. I have seen trees fallen on power lines and the lines have been almost touching the ground (of course, the poles were bent too). Are you sure that the birds were making it sag? Power lines always droop a bit. --T H F S W (T · C · E) 22:09, 29 November 2010 (UTC)[reply]
To answer question 2) Engineers who design these things are not complete idiots. Birds are known to congregate on power lines, and I am certain that the strength of the line is designed to take this into account. --Jayron32 04:11, 30 November 2010 (UTC)[reply]
I believe there was a documentary produced about ten years ago that studied this problem in depth. Wikipedia has an article about it if you're intererested. —Bkell (talk) 12:52, 30 November 2010 (UTC)[reply]
Ha, ha, Bkell, very nice connection.
Seriously, overhead power lines are constructed to some pretty amazing engineering standards. I don't know if bird loads are ever specifically accounted for, but in areas where it freezes, the standard is that the line must be able to support the weight of an ice coating up to five times the wire's diameter. In areas subject to extreme ice storms -- such as Quebec -- the standard is raised to ten times. (And yet, they are still occasionally pulled down due to stupendous quantities of encased ice. [30])
One of the most amazing demonstrations of the "hoisting capacity" of an overhead power line occurred in Seattle in April, 1998, when a small private airplane got tangled in one and dangled from it for a couple of hours [31] [32] before some very intrepid firefighters effected a one-of-a-kind rescue. [33]Steve Summit (talk) 13:20, 30 November 2010 (UTC)[reply]
Not all power lines are the same. The one which supported the plane was 50 feet off the ground and supplied a steel mill and 2500 customers. It was likely a transmission or subtransmission high voltage line, and had little in common with the line carrying power to ones house from the transformer, or in many cases carrying power to the local transformer. The high lines may be steel reinforced, while the line in the alley or along the highway in front of your house may be far weaker. A 4kv or 12kv feeder might be relatively small copper wire. The service going from the transformer to a house might be rather small as well. Even so, it is hard to envision bird loading sufficient to break wire that stands up to wind and ice. Edison (talk) 17:20, 30 November 2010 (UTC)[reply]

Fluorescent light flashing after being switched off

The energy-saving fluorescent light in my kitchen has just behaved rather strangely. Just now I switched it off, after which it flashed at intervals of approximately 1 second, for a total time of about a minute. Right at the end of this just one of the three "loops" was flashing. What could cause this? Thanks, DuncanHill (talk) 22:15, 29 November 2010 (UTC)[reply]

The modern Compact fluorescent lamp have voltage control circuitry (the old ones chokes) so that they can run between 230 -ish volts and about 256 -ish volts. Once the power is off, it might just be the capacitors discharging. The reserved charge is dissipated each time the arc is achieved. After a few flickers the capacitors are completely discharges. Also, maybe you lighting loops are wired in the 'live' configuration. Nothing wrong with this (according to the UK regs) but important that you make sure the power's off before allowing you pinkies any where near the copper.--Aspro (talk) 22:42, 29 November 2010 (UTC)[reply]
Oh! And is your kitchen very cold. Dry air and low temperatures allow florescent tubes to exhibit behaviours not seen at higher temperatures. --Aspro (talk) 22:45, 29 November 2010 (UTC)[reply]
Has been very cold lately, and I think there's cold air coming into the flat above. DuncanHill (talk) 23:09, 29 November 2010 (UTC)[reply]
I asked this question just the other day! Zunaid 13:36, 30 November 2010 (UTC)[reply]

Antonov AN225- Personal experience and photographs available

Hi there,I flew on the AN 225 from Manchester in 2006, and have a little knowledge and plenty of photographs of it should they be of interest? —Preceding unsigned comment added by Filler9 (talkcontribs) 23:13, 29 November 2010 (UTC)[reply]

Please consider licensing your images and uploading them to Wikimedia Commons. Read Wikipedia:Uploading images and be sure you understand the implications of licensing your images; then use the uploading form to put them up. Also check out our article, and feel free to edit it to add content; but be sure you understand our verifiability policy and do not contribute original research. Nimur (talk) 23:21, 29 November 2010 (UTC)[reply]
(ec) Please consider improving the Antonov An-225 article and/or uploading your photos to the commons:Antonov An-225 Wikimedia Commons category. Ginger Conspiracy (talk) 23:22, 29 November 2010 (UTC)[reply]

Position of maximum depression on classical guitar top.

At present I am in the process of writing an article on the construction of the classical guitar, and am proposing a different method of reinforcing the top-plate. The method consists of a set of bars, radiating from a position in front of the bridge, which is where maximum deflection occurs, due to the action of the strings when plucked. Where that position is, can be calculated, I believe, with a formula to do with Modulus of Elasticity, when the following is known: The type of wood and thickness of the top-plate. The total tensile and torsional force applied to the bridge, from the strings. The edge pressure from the bridge. This little information is gained from the mechanical drafting certificate course I did more than 40 years ago, hence the sketchy description. —Preceding unsigned comment added by 58.175.87.33 (talk) 04:27, 30 November 2010 (UTC)[reply]

And your question is . . . ? 87.81.230.195 (talk) 14:25, 30 November 2010 (UTC)[reply]
We do have articles on Classical guitar and Classical guitar_making -- are they helpful? --- Medical geneticist (talk) 21:41, 30 November 2010 (UTC)[reply]
How accurate do you want the results? There is no shortage of finite element analysis applied to guitar body: Modal analysis of an acoustic guitar by finite element came up in a web search; and you can find even more scholarly, technical, and accurate modeling with a little effort. Obviously, the specific node point of maximum vibration amplitude is going to depend on the guitar body shape, materials, and how accurately your mathematical model matches physical reality. Nimur (talk) 21:49, 30 November 2010 (UTC)[reply]

Relativity questions

Expand long text

The Phenomenon of Far Event Dilation

For a given event eA happened at time tA and location A, an observer at location O uses a camcorder with timer to record event eA, then the recorded event time ta will always behind tA, this is the phenomenon of Far Event Dilation. This is prepared for "Distance Relativity", please comment.

1. The Equation of Far Event Dilation The recorded event time ta can be calculated by ta=tA+(AO/c’) ---(1), c’ is the speed of light in the environment.

2. The Equation of Event Period If the event ends at location B and time tB, then the recorded time period can be calculated by (tb-ta)=(tB-tA)+((BO-AO)/c’) ---(2).

2-1. When BO=AO, that means, if A and B are located on the same sphere with center O, then, the recorded event period is always the same as the actual time period.

2-2. When BO>AO, that means, the event ends at a point farther away, then the recorded event period is larger than the actual event period. The event looks happening slower in the video.

2-3. When BO<AO, that means, the event ends at a point closer to O, then the recorded event period is smaller than the actual event period. The event looks happening quicker in the video.Jh17710 (talk) 06:27, 30 November 2010 (UTC)[reply]

Distance Relativity

Now, let us study the Far Event Dilation in one dimension space. We assume the event points A or B and the location O of the observer are always on the same line. We also focus on cases that either the event moves or the observer moves at constant velocity v. Then we will have two sets of time equations and one set of velocity equations. This is the main subject, please comment.

3. When observer moves, then |BO-AO|=v(tb-ta)

3-1. When BO>AO, (tb-ta)=(c’/(c’-v))(tB-tA) ---(3)

3-2. When BO<AO, (tb-ta)=(c’/(c’+v))(tB-tA) ---(4)

4. When event moves, then |BO-AO|=v(tB-tA)

4-1. When BO>AO, (tb-ta)=((c’+v)/c’)(tB-tA) ---(5)

4-2. When BO<AO, (tb-ta)=((c’-v)/c’)(tB-tA) ---(6)

5. Let the event be the motion of an object from A to B under the constant velocity of v and let the speed of that object be calculated as V by the observer, then:

5-1. When BO>AO, V=(c’/(c’+v))v ---(7), particularly when v=c’, we have V=c’/2.

5-2. When BO<AO, V=(c’/(c’-v))v ---(8), when v=c’, we find out V is infinitely fast. For example, if a base ball flies to me at the speed of c’ and when it passes the sign of 30 feet, that particular picture will take about 100 nanoseconds to reach my eyes, but, at around 99 nanoseconds, that base ball already hits my nose so that the speed of that base ball is unlimited fast to me. Actually, all photons run into our eyes at infinite fast speed.Jh17710 (talk) 06:27, 30 November 2010 (UTC)[reply]

The Phenomenon of Far Ball Contraction

We can calculate the distance of a ball by the equation based on the phenomenon of Far Ball Contraction, which is “A ball looks smaller when it is farther away.” This is just to show the fact that we cannot see the dimension representing far and near, we calculate that dimension.

6. Far Ball Contraction Formula If we hold a ruler 1 foot away from our eyes and measure a ball of radius r feet at distance of L feet away from our eyes to get radius of R feet, then, when the ball moves to another distance of L’ feet, the measured radius R’ feet will be R’=(RL)/L’ ---(9). We should keep both of L and L’ larger than (1+2r) feet to make the measuring job practical.

7. When the Ball is Replaced by a Brick If we replace the far ball by a Brick and look at the length and height side of that brick, then, we don’t know how wide that brick is. That means, we judge the distance of an object by the image of the length and height of the object that is perpendicular to our sight line, and normally, we have no way to tell how wide or how deep that object is.Jh17710 (talk) 06:27, 30 November 2010 (UTC)[reply]

Response

Jh17710: People who patrol the reference desk aren't going to answer homework questions for you; also we generally need questions to answer. It looks above like you have copied large swaths of text from some physics text somewhere. Is there a question anywhere in all of this? --Jayron32 06:48, 30 November 2010 (UTC)[reply]

... and the point that the text seems to be trying to make appears false to me. For both ball and brick, we infer both distance and depth from past experience, and if we have doubt about our guess, we use parallax to check. The idea of "infinite speed" is also wrong. Have you read our article on Special relativity? Dbfirs 07:41, 30 November 2010 (UTC)[reply]
I should also note that you're getting close to where fair use may give way to copyright violation, unless you're actually going to discuss bits and pieces of all that stuff in detail. Wnt (talk) 10:44, 30 November 2010 (UTC)[reply]

is it possible to blow up a planet like in Star Wars?

Lets say there were a really big gun or energy beam aimed at a planet the size of Earth? Could it actually cause the Earth to blow up, or would it just cause earthquakes and cause the rock to melt and things like this? I am not talking about an impact event, this is like something worse than that, like the Death Star, pretty much. But would the laws of phsics allow a planet to EXPLODE? is it even posible? —Preceding unsigned comment added by The Fat Man Who Never Came Back (talkcontribs) 03:42, November 30, 2010

Sure, it's possible in some sense, but not like in Star Wars. All in all, you should realize this planet is basically a lump of metal several thousand miles in diameter, stuff that likes to congregate (gravity) and will not easily be scattered into a gazillion pieces. The energy required to actually blow it up as you describe would be an immense amount.--Rallette (talk) 09:00, 30 November 2010 (UTC)[reply]
Someone worked out details on what you'll need to do it here. Have fun! --Sean 14:22, 30 November 2010 (UTC)[reply]

What would happen if scientists blew up the moon?

What would be the consequences here on Earth if we blew up the moon? I imagine there would be all sorts of environmental disasters, but would there be any benefits?--The Fat Man Who Never Came Back (talk) 08:44, 30 November 2010 (UTC)[reply]

Some of this was discussed at Wikipedia:Reference_desk/Archives/Science/2006_November 7#Blowing up the moon and Wikipedia:Reference_desk/Archives/Science/2007 May 6#Earth's Spin. Wnt (talk) 10:47, 30 November 2010 (UTC)[reply]
This is one advantage. Paul (Stansifer) 14:41, 30 November 2010 (UTC)[reply]
This is another. ;) --Link (tcm) 19:47, 30 November 2010 (UTC)[reply]
You attribute some pretty impressive powers to these 'scientists'. In any event, the answer depends on what you mean by 'blew up'. If you just fracture it without giving any of the chunks lunar escape velocity, then they'll pretty much settle back into a new Moon. (The surface features won't be the same, obviously, but if the whole system stays gravitationally bound then you'll eventually get a Moon-sized approximately-spherical lump back.) If you manage to compress enough of it into a wee tiny black hole (bigger than CERN LHC-sized, so it won't evaporate immediately) it will eventually all collapse into a stable Moon-massed black hole (see Roger MacBride Allen's The Ring of Charon). In either case, not a whole lot happens on Earth. The former case generates an impressive light show, and the latter case makes it a bit darker at night, but tides won't be appreciably affected and the Earth will carry on essentially unchanged.
If you redistribute the mass of the Moon (as by pulverizing it into a ring in lunar orbit), or remove the Moon and all its mass entirely by some yet to be discovered magical phenomenon (or by virtue of it actually being a camouflaged alien spaceship, see David Weber's Mutineers' Moon) then you have serious effects on Earth. Getting rid of tides (well, most tides — there will still be miniscule tides from the Sun's attraction) will screw over a lot of coastal life. Depending on how quickly you remove or re-adjust the Moon's position, you might also get some serious earthquakes going on, as rapidly removing the Moon's pull on Earth may have the effect of 'twanging' crust.
If you use some unimaginably violent explosion to 'blow up' the Moon in what might be the most conventional picture of such an event, you're very likely to have some Texas-size chunks hit the Earth. When that happens, we have a massive winter, and most life on Earth dies. What benefits did you have in mind? TenOfAllTrades(talk) 14:45, 30 November 2010 (UTC)[reply]
Here's a fun article on the subject : The Straight Dope:I plan to destroy the moon. What effect would this have on the earth?
APL (talk) 15:37, 30 November 2010 (UTC)[reply]
Related question: does humanity at present time have the capacity to blow the Moon to smithereens? Bus stop (talk) 15:45, 30 November 2010 (UTC)[reply]
Certainly not. --Sean 18:51, 30 November 2010 (UTC)[reply]
All this and yet nobody mentions the late Alexander Abian. --Anonymous, 23:24 UTC, November 30, 2010.

Can we blow up comets?

A few years ago, we tried and failed to blow up a comet. We shot a missile at it, and it caused an explosion and released a dust cloud, but unfortunately the comet was not destroyed. In the future, if we shoot more powerful projectiles at comets, is it possible to destroy them altogether? What would happen to the galaxy if we eliminated all comets and meteors and asteroids?--The Fat Man Who Never Came Back (talk) 08:48, 30 November 2010 (UTC)[reply]

I think you've read the article wrong if you think the Deep Impact space mission was an attempt to destroy a comet. Nil Einne (talk) 10:53, 30 November 2010 (UTC)[reply]
If you blow it up, you just seperate a mass into smaller bits of mass. Each of the bits are still comets, just smaller. So nothing would "happen to the galaxy", except it would have smaller comets. --Lgriot (talk) 12:15, 30 November 2010 (UTC)[reply]
As has been hinted at above you should read the galaxy article to adjust your mental image of the scale of things.
It sounds like you may have meant solar system instead of 'galaxy'. If so, have a read of the Oort cloud article to get an idea of what sized job ridding the solar system of all comets would be. Blakk and ekka 12:33, 30 November 2010 (UTC)[reply]
All the comets in the Solar System make up the mass of about 5 Earths, and less than 1% of the mass of the Solar System as a whole. There's nothing you can do to a comet to have much effect on the Solar System, much less the galaxy. --Sean 14:25, 30 November 2010 (UTC)[reply]

Which organelles contain DNA?

I always thought it was just the nucleus & either mitochondria (in animals) or chloroplasts (in plants). However, I found a passage in Lewis Thomas's "The Lives of a Cell" where he intimates that this property is shared by other organelles too:

Mitochondria are stable and responsible lodgers, and I choose to trust them. But what of the other little animals, similarly established in my cells, sorting and balancing me, clustering me together? My centrioles, basal bodies, and probably a good many other more obscure tiny beings at work inside my cells, each with its own special genome, are as foreign, and as essential, as aphids in anthills.

Is this an inaccuracy? The book was written in 1974. Thanks. AGradman / how the subject page

at 12:19, 30 November 2010 (UTC)

None of these sub-cellular bodies have their own genome. It sounds like Thomas may be putting forward a variation of Endosymbiotic theory. Blakk and ekka 12:49, 30 November 2010 (UTC)[reply]
I'm not sure if it was just a poor choice of words or genuine confusion but do note nearly all eukaryotes have mitochondrion, including plants. Nil Einne (talk) 13:53, 30 November 2010 (UTC)[reply]
In the case of centrioles, it may have been based on outdated research. From centrosome:
Research in 2006 indicated that centrosomes from Surf clam eggs contain RNA sequences. The sequences identified were found in "few to no" other places in the cell, and do not appear in existing genome databases. One identified RNA sequence contains a putative RNA polymerase, leading to the hypothesis of an RNA based genome within the centrosome. However, subsequent research has shown that centrosome do not contain their own DNA-based genomes. While it was confirmed that RNA molecules associate with centrosomes, the sequences have still been found within the nucleus. Furthermore, centrosomes can form de novo after having been removed (e.g. by laser irradiation) from normal cells.
--Sean 14:29, 30 November 2010 (UTC)[reply]

velocity of sound

why does the velocity of sound in water decrease after reaching a maximum velocity at a certain temperature —Preceding unsigned comment added by 41.221.209.6 (talk) 13:46, 30 November 2010 (UTC)[reply]

The effect I'm familiar with is that as a function of depth, the sound speed increases after reaching a minimum at a certain depth (which turns out to be temperature-related). Our Speed of sound article shows the effect but doesn't explain it well and explains it briefly. There's a better fuller discussion at DOSITS. In a nutshell: the speed of sound in seawater is proportional to temperature, to pressure, and to depth. Temperature decreases as depth decreases, which is why the sound speed initially decreases as you descend below the surface. But then, of course, pressure begins to increase, and so the sound speed goes back up as you go really deep. —Steve Summit (talk) 14:12, 30 November 2010 (UTC), tweaked 14:48, 30 November 2010 (UTC)[reply]

Daytime flash dreams

Sometimes I tend to experience ultra-short (for about 0,5 sec) unconscious flash dreams while not sleeping, just resting. They look like normal night dreams, but appear even when my eyes are open. Most recently for example I saw a nice red motorcycle with bald biker. Is there a name for such phenomenon? —Preceding unsigned comment added by 89.77.158.172 (talk) 17:01, 30 November 2010 (UTC)[reply]

Well, this is not really a medical question and is quite common, so see daytime parahypnagogia--Aspro (talk) 17:09, 30 November 2010 (UTC)[reply]

Margarine and oils without saturated fat

Are there any brands or kinds of margarines and oils with little saturated fat? Even the "Light" margarine I have in front of me is over 9% saturated fat. Thanks 92.28.247.40 (talk) 17:26, 30 November 2010 (UTC)[reply]

Shortenings like margarine and crisco are just vegetable oil that's been "partially hydrogenated". Saturation is one of the main reasons these things are solid at room temperature. The unsaturated fat is lowers the melting temperature to make vegetable oil a liquid). The lowest saturated-fat content I see on that article's table is canola oil at 7%, and all the solid ones are somewhat higher. DMacks (talk) 17:34, 30 November 2010 (UTC)[reply]
In general the more saturated the oil the lower the melting point. So unsaturated oils are liquid at room temperature. However the length of the molecule (called the chain length) also plays a role, the shorter the chain the lower the melting point. As far as health goes just looking at saturated vs unsaturated is not enough, there is plenty of evidence that certain chain lengths are much unhealthier. In general longer chains are less healthy. So you can have an unsaturated, long chain oil that is less healthy that a short chain, but saturated oil. This is the main reason people switched to saturated palm oils, they might be saturated, but their oil has few bad health effects, and some believe actually improve health! In general it's best to eat oils that are commonly found in food. The reason is that the health effects of the oil are directly related to how well the body can "burn" the oil (lipid) for fuel. To burn the lipid requires enzymes, and the body has more enzymes for some types than for others. Another thing, Essential fatty acids (EFA) are the best type of lipid to eat, but they are all liquid and polyunsaturated. A manufacturer has a choice - they can make the margarine with some saturated fat, and a little unsaturated, or they can make it with polyunsaturated and more saturated (the high and low melting points cancel each other out). Which is better? You are getting more saturated fat, but also more EFAs, or less saturated fat, but no EFAs. My feeling is that the version with saturated, but also polyunsaturated is better, but the exact answer on the proper balance requires lots of studies that as far as I know have not been done. A final point is that I think it's better to eat a margarine with a variety of fat types in it, vs. one that is just one kind of lipid - even if you have to eat some saturated fat in the process. Whew, this post was longer than I expected, I hope it helps you. Ariel. (talk) 19:17, 30 November 2010 (UTC)[reply]
I cannot see the answer to my question in there, but thanks for trying. 92.28.247.40 (talk) 20:07, 30 November 2010 (UTC)[reply]
Basically what he's? saying is 1) It's rather unlikely there are any margarines without saturated oils. 2) If your concerned about health, just avoiding all saturated fats likely isn't the way to go (technically you didn't ask about this but given your question I don't think it was an unresonable response). Note that considering no 2 even if someone could make an oil without saturated fats, it's not likely they would. According to [34] one of the lowest saturated fat oils is enova brand but um [35]... In any case it sounds like weird stuff [36] P.S. It's also possible Ariel missed your point about oils at first since you referred to the saturated fat content in margarine rather then oils. I did. Nil Einne (talk) 20:30, 30 November 2010 (UTC)[reply]

Bioluminescence

hi, Today I saw (in a video) a creature which made a Bioluminescence. It looked like fireworks. the liquid which he spitted (it was a marine animmal) had glown for 3 seconds,and then turn-off. and then, like a lightning it glew powerfully. does anyone know what that sea animmal's name? 84.228.119.63 (talk) —Preceding undated comment added 17:51, 30 November 2010 (UTC).[reply]

Well. There is this one, and as ever, Wikipedia has an article on it:Heterocarpus--Aspro (talk) 19:41, 30 November 2010 (UTC)[reply]

We also have Bioluminescence of course which lists a bunch of animals Nil Einne (talk) 21:32, 30 November 2010 (UTC)[reply]

Aggression, dementia, disinhibition

1) Some elderly people seem to be habitually hostile - are there any other things apart from dementia that cause this aggression in some elderly? 2) Does having dementia make people disinhibited? Thanks 92.28.247.40 (talk) 18:00, 30 November 2010 (UTC)[reply]

When any disease or injury effects the normal functioning of the pre-frontal cortex and especially when executive functions with in them are effected, it can result in more aggressive behaviour and/or dis-inhibition than would normally expected. Dementia does not always lead to more aggression but is more common than not. --Aspro (talk) 18:21, 30 November 2010 (UTC)[reply]

Ok, so aggression is a common result of dementia, but does being aggressive and elderly suggest dementia? Thanks. 92.28.247.40 (talk) 22:18, 30 November 2010 (UTC)[reply]

On its own - no. In combination with other things - maybe. Try the alzheimers-research.org.uk website for some more information. --Tagishsimon (talk) 22:22, 30 November 2010 (UTC)[reply]
Wouldn't hurt to check for it, but some old people can just be mean ol' bastards. HalfShadow 22:25, 30 November 2010 (UTC)[reply]

Why must some truffles be found and not grown?

This MSNBC article says that Piedmont white truffles cost around US$3,000 per pound because of low supply, and also claims "this is one food that can’t be formulated in a lab or grown in a greenhouse." Why? By now there must have been attempts to farm these truffles in order to try to cash in on this. What were the results? Comet Tuttle (talk) 19:02, 30 November 2010 (UTC)[reply]

That is a misleading statement. Truffle farms are, essentially, tree farms. It takes about 30 years to grow the trees (and the truffles). So, that statement could be true of any large tree. For example, you could claim that Christmas Trees are the one decoration that can't be formulated in a lab or grown in a greenhouse. What you actually mean is that the tree is big. You need a lot of room. So, for a lot of trees, you'd need an enormous greenhouse. Obviously, you'd use a field instead - which is what truffle farms use, large fields. -- kainaw 19:09, 30 November 2010 (UTC)[reply]
(after e/c)
Truffles grow on the roots of Oak trees. Perhaps they simply mean that truffles need vast orchards to cultivate, even though the truffles themselves are rather small? (Literally, adult oak trees are unlikely to grow in either a lab or all but the largest greenhouses.) APL (talk) 19:11, 30 November 2010 (UTC)[reply]
I see; thank you. I had jumped to the conclusion that finding truffles was a hunter/gatherer task and a crapshoot, rather than an organized and planned farming process. Comet Tuttle (talk) 19:26, 30 November 2010 (UTC)[reply]
Note that unsurprisingly our article has a section on cultivation Truffle (fungus)#Cultivation Nil Einne (talk) 20:34, 30 November 2010 (UTC)[reply]